Решить модульное уравнение: Как решать уравнения с модулем

Содержание

Уравнения с модулем. Исчерпывающий гид (ЕГЭ — 2021)

4. Для каждого интервала запишем и решим уравнение. Важно проследить, чтобы ответы соответствовали интервалу!

Примеры:

I. \(\displaystyle x<-3\).

Здесь оба модуля раскрываем «с минусом»:

\(\displaystyle-\left( x+3 \right)+\left( 2{x}-1 \right)=1\text{ }\Leftrightarrow \text{ }-{x}-3+2{x}-1=1\text{ }\Leftrightarrow \text{ }x=5\text{ }>-3\) – этот корень сторонний.

II. \(\displaystyle-3\le x<\frac{1}{2}\).

Здесь первый модуль раскрываем «с плюсом», а второй – «с минусом»:

\(\displaystyle\left( x+3 \right)+\left( 2{x}-1 \right)=1\text{ }\Leftrightarrow \text{ }x+3+2{x}-1=1\text{ }\Leftrightarrow \text{ }x=-\frac{1}{3}\) – этот корень попадает в «свой» интервал, значит, он подходит.

III. \(\displaystyle x\ge \frac{1}{2}\).

Здесь оба модуля раскрываем «с плюсом»:

\(\displaystyle\left( x+3 \right)-\left( 2{x}-1 \right)=1\text{ }\Leftrightarrow \text{ }x+3-2{x}+1=1\text{ }\Leftrightarrow \text{ }x=3\) – этот корень тоже является решением.

Проверим полученные корни:

I. \(\displaystyle x=5:\text{ }\left| 5+3 \right|-\left| 2\cdot 5-1 \right|=8-9=-1\ne 1\) (корень и правда сторонний).

II. \(\displaystyle x=-\frac{1}{3}:\text{ }\left| -\frac{1}{3}+3 \right|-\left| 2\cdot \left( -\frac{1}{3} \right)-1 \right|=\frac{8}{3}-\frac{5}{3}=1\).

III. \(\displaystyle x=3:\text{ }\left| 3+3 \right|-\left| 2\cdot 3-1 \right|=6-5=1\).

Ответ: \(\displaystyle-\frac{1}{3};\text{ }3.\)

Примеры:

Решения:

1. \( \displaystyle \left| x+2 \right|-\left| 3{x}-1 \right|+\left| 4-x \right|=3\)\( \displaystyle \left[ \begin{array}{l}x+2=0\text{  }\Rightarrow \text{  }x=-2\\3{x}-1=0\text{  }\Rightarrow \text{ }x=\frac{1}{3}\\4-x=0\text{  }\Rightarrow \text{  }x=4\end{array} \right.\) 

I. \( \displaystyle -2\le x<\frac{1}{3}.\)

\( \displaystyle -\left( x+2 \right)+\left( 3{x}-1 \right)+\left( 4-x \right)=3\)

\( \displaystyle x=2>-2\Rightarrow \) – корень сторонний

II. \( \displaystyle -2\le x<\frac{1}{3}\)

\( \displaystyle \left( x+2 \right)+\left( 3{x}-1 \right)+\left( 4-x \right)=3\Leftrightarrow \)

\( \displaystyle 3x=-2\Leftrightarrow x=-\frac{2}{3}\in \left[ -2;\frac{1}{3} \right)\) – подходит

III. \( \displaystyle \frac{1}{3}\le x<4\)

\( \displaystyle \left( x+2 \right)-\left( 3{x}-1 \right)+\left( 4-x \right)=3\Leftrightarrow \)

\( \displaystyle -3x=-4\Leftrightarrow x=\frac{4}{3}\in \left[ \frac{1}{3};4 \right)-\) подходит

IV. \( \displaystyle x\ge 4\)

\( \displaystyle \left( x+2 \right)-\left( 3{x}-1 \right)-\left( 4-x \right)=3\Leftrightarrow \)

\( \displaystyle x=4\Leftrightarrow x=-4<4\text{ }-\) корень сторонний

Ответ\( -\frac{2}{3};\text{  }\frac{4}{3}.\)

2. \( \left| 3{x}-5 \right|+\left| 3+2x \right|=2\left| x+1 \right|\text{  }\Leftrightarrow \text{  }\left| 3{x}-5 \right|+\left| 3+2x \right|-2\left| x+1 \right|=0.\)

\( \left[ \begin{array}{l}3{x}-5=0\text{  }\Rightarrow \text{ }x=\frac{5}{3}\\3+2x=0\text{  }\Rightarrow \text{ }x=-\frac{3}{2}\\x+1=0\text{    }\Rightarrow \text{  }x=-1\end{array} \right.\)

I. \( \displaystyle x<-\frac{3}{2}\)

\( \displaystyle -\left( 3{x}-5 \right)-\left( 3+2x \right)+2\left( x+1 \right)=0\Leftrightarrow \)

\( \displaystyle -3x=-4\Leftrightarrow x=\frac{4}{3}>-\frac{3}{2}\Rightarrow \) корень сторонний

II. \( \displaystyle -\frac{3}{2}\le x<-1\)

\( \displaystyle -\left( 3{x}-5 \right)+\left( 3+2x \right)+2\left( x+1 \right)=0\Leftrightarrow \)

\( \displaystyle x=-10<-1\Rightarrow \) корень сторонний

III. \( \displaystyle -1\le x<\frac{5}{3}\)

\( \displaystyle -\left( 3{x}-5 \right)+\left( 3+2x \right)-2\left( x+1 \right)=0\Leftrightarrow \)

\( \displaystyle -3x=-6\Leftrightarrow x=2\text{  }>\frac{5}{3}\Rightarrow \) корень сторонний

IV. \( \displaystyle x\ge \frac{5}{3}\)

\( \displaystyle \left( 3{x}-5 \right)+\left( 3+2x \right)-2\left( x+1 \right)=0\Leftrightarrow \)

\( \displaystyle 3x=4\Leftrightarrow x=\frac{4}{3}<\frac{5}{3}\Rightarrow \) корень сторонний

Итак, ни на одном интервале не нашлось корней. Значит, решений это уравнение не имеет.

Ответ: Решений не имеет.

1 x модуль

Вы искали 1 x модуль? На нашем сайте вы можете получить ответ на любой математический вопрос здесь. Подробное решение с описанием и пояснениями поможет вам разобраться даже с самой сложной задачей и 1 модуль x, не исключение. Мы поможем вам подготовиться к домашним работам, контрольным, олимпиадам, а так же к поступлению в вуз. И какой бы пример, какой бы запрос по математике вы не ввели — у нас уже есть решение. Например, «1 x модуль».

Применение различных математических задач, калькуляторов, уравнений и функций широко распространено в нашей жизни. Они используются во многих расчетах, строительстве сооружений и даже спорте. Математику человек использовал еще в древности и с тех пор их применение только возрастает. Однако сейчас наука не стоит на месте и мы можем наслаждаться плодами ее деятельности, такими, например, как онлайн-калькулятор, который может решить задачи, такие, как 1 x модуль,1 модуль x,1 модуль x 2,1 модуль x 3,1 модуль х,2 модуль x,2 модуль х,2 х модуль,2х 3 5 модуль,3 модуль x,3 модуль х,4 x 5 модуль,4 модуль х,5 модуль,5 модуль x,5 модуль х,7 класс уравнения модулями с,f x модуль x,x 2 модуль,x 3 модуль,x 5 модуль,x модуль,x модуль 2,y модуль 1 x 1,выражения с модулем,действия с модулем,действия с модулями,задания с модулем,задачи с модулем,задачи с модулями,икс модуль,как избавиться от модуля,как модуль умножить на модуль,как раскрывается модуль,как раскрывать модули,как раскрывать модуль,как раскрывать модуль в уравнении,как раскрыть модуль в уравнении,как решается модуль,как решать квадратные уравнения с модулем,как решать модули,как решать модуль,как решать модуль в модуле,как решать модуль равен модулю,как решать модульные уравнения,как решать модульные уравнения 7 класс,как решать примеры с модулем,как решать примеры с модулями,как решать с модулем,как решать уравнение с двойным модулем,как решать уравнение с модулем,как решать уравнение с модулем 7 класс,как решать уравнение с модулями,как решать уравнения 6 класс с модулями,как решать уравнения с двойным модулем,как решать уравнения с двумя модулями,как решать уравнения с модулем,как решать уравнения с модулем 10 класс,как решать уравнения с модулем 7 класс,как решать уравнения с модулем 9 класс,как решать уравнения с модулями,как решать уравнения с модулями 10 класс,как решать уравнения с модулями 7 класс,как решаются модули,как решаются уравнения с модулем,как решаются уравнения с модулями,как решить квадратное уравнение с модулем,как решить модуль,как решить модуль в модуле,как решить модульное уравнение,как решить уравнение квадратное с модулем,как решить уравнение с двумя модулями,как решить уравнение с модулем,как решить уравнение с модулем 7 класс,как решить уравнение с модулями,как решить уравнения с модулем,как убрать модуль в уравнении,как умножить модуль на модуль,калькулятор модулей уравнений,калькулятор решение уравнений с модулем,калькулятор уравнений с модулем,калькулятор уравнений с модулями,калькулятор уравнений с модулями онлайн,калькулятор уравнения с модулем,квадратное уравнение с модулем,квадратные уравнения с модулем,квадратные уравнения с модулем как решать,линейные уравнения с модулем,минус модуль х равен минус х решить,модули как раскрывать,модули как решать,модули как решаются,модули примеры,модули решение,модули решение уравнений,модули уравнения,модуль 1 x,модуль 1 х,модуль 1 х больше 2,модуль 2 x,модуль 2 х,модуль 2 х 3,модуль 3 x,модуль 3 равен х,модуль 3 х,модуль 4 х,модуль 5 x 4,модуль 5 х,модуль x,модуль x 1,модуль x 1 3,модуль x 2,модуль x 2 3,модуль x 3,модуль x 4,модуль x 4 3,модуль x 4 x,модуль x 5,модуль x 5 x,модуль x равен,модуль x равен x,модуль в модуле,модуль в модуле как решать,модуль в модуле как решить,модуль в модуле решение,модуль в модуле уравнение,модуль в уравнении как раскрыть,модуль в уравнениях,модуль выражения,модуль икс,модуль икс равен икс,модуль как раскрыть,модуль как решается,модуль как решать,модуль как решить,модуль квадратного уравнения,модуль минус икс,модуль минус икс равен икс,модуль плюс модуль равно модуль,модуль примеры,модуль примеры решения,модуль равен 2,модуль равен x,модуль равен модулю как решать,модуль равен модулю уравнение,модуль раскрыть,модуль решение,модуль решение уравнений,модуль уравнение,модуль уравнения,модуль х,модуль х 1,модуль х 1 х 3,модуль х 1 х 3 1,модуль х 2,модуль х 2 5,модуль х 3,модуль х 3 2,модуль х 4,модуль х 4 х,модуль х 5,модуль х 5 2,модуль х 8 5,модуль х минус х,модуль х модуль у 1,модуль х модуль у 3,модуль х равен 3,модуль числа решение уравнений,модуль числа уравнения,модульное уравнение,модульное уравнение решить онлайн,модульные уравнения,модульные уравнения 10 класс,модульные уравнения 7 класс,модульные уравнения 7 класс как решать,модульные уравнения как решать,модульные уравнения решение,модуля решение,онлайн раскрытие модуля,онлайн решение модулей,онлайн решение модульных уравнений,онлайн решение уравнение с модулем,онлайн решение уравнений с модулем,онлайн решение уравнений с модулем с подробным решением,онлайн решение уравнений с модулями,онлайн решение уравнения с модулем,онлайн решить уравнение с модулем,онлайн решить уравнения с модулем,онлайн уравнения с модулем,правила модуля,правила раскрытия модуля,правило модуля,правило раскрытия модуля,примеры как решать модули,примеры модули,примеры модуль,примеры решения квадратные уравнения с модулем,примеры с модулем,примеры с модулем как решать,примеры с модулями,примеры с модулями 7 класс,примеры с модулями как решать,примеры с модулями примеры решений,простейшие уравнения с модулем,равен модуль 2,раскрытие модулей,раскрытие модуля,раскрытие модуля в уравнении,раскрытие модуля онлайн,раскрыть модуль,раскрыть модуль онлайн,решение задач с модулем,решение квадратных уравнений с модулем,решение линейных уравнений с модулем 7 класс примеры,решение модулей,решение модулей онлайн,решение модули,решение модуль в модуле,решение модульные уравнения,решение модульных уравнений,решение модульных уравнений 7 класс,решение модульных уравнений онлайн,решение модуля,решение онлайн модулей,решение примеров с модулем,решение примеров с модулями,решение с модулем,решение уравнение онлайн с модулем,решение уравнение с модулем,решение уравнение с модулем онлайн,решение уравнений модули,решение уравнений модуль,решение уравнений модуль числа,решение уравнений онлайн с модулем,решение уравнений онлайн с модулями,решение уравнений онлайн с подробным решением с модулем,решение уравнений с двойным модулем,решение уравнений с двумя модулями,решение уравнений с модулем,решение уравнений с модулем 7 класс,решение уравнений с модулем 7 класс примеры,решение уравнений с модулем калькулятор,решение уравнений с модулем онлайн,решение уравнений с модулем онлайн с подробным решением,решение уравнений с модулем с подробным решением,решение уравнений с модулем с подробным решением онлайн,решение уравнений с модулями,решение уравнений с модулями онлайн,решение уравнения онлайн с модулем,решение уравнения с модулем,решение уравнения с модулем онлайн,решение уравнения с модулем онлайн калькулятор,решения уравнений с модулем,решения уравнений с модулями,решите уравнение с модулем,решить модульное уравнение онлайн,решить онлайн уравнение с модулем,решить уравнение модуль х равен минус х,решить уравнение модуль х равен х,решить уравнение онлайн с модулем,решить уравнение с модулем,решить уравнение с модулем онлайн,решить уравнение с модулем онлайн с решением,решить уравнения онлайн с модулем,решить уравнения с модулем онлайн,рівняння з модулем,рівняння з модулями,с двумя модулями уравнение,сложные уравнения с модулем,у 2 модуль х,у 3 модуль х,у модуль х 2,уравнение модуль,уравнение модуль в модуле,уравнение модуль равен модулю,уравнение с двойным модулем как решать,уравнение с двумя модулями,уравнение с модулем,уравнение с модулем 7 класс,уравнение с модулем как решать,уравнение с модулем как решать 7 класс,уравнение с модулем квадратное,уравнение с модулем квадратное уравнение,уравнение с модулем онлайн,уравнение с модулем онлайн решение,уравнение с модулем примеры,уравнение с модулем решение,уравнение с модулем решение онлайн,уравнение с модулями,уравнение с модулями 7 класс,уравнение с модулями как решать,уравнения в модуле,уравнения модули,уравнения модуль,уравнения модуль числа,уравнения онлайн с модулем,уравнения с двойным модулем как решать,уравнения с двумя модулями,уравнения с двумя модулями как решать,уравнения с модулем,уравнения с модулем 10 класс как решать,уравнения с модулем 7 класс,уравнения с модулем 7 класс примеры решения,уравнения с модулем 8 класс примеры решения,уравнения с модулем как решать,уравнения с модулем как решать 7 класс,уравнения с модулем как решить,уравнения с модулем калькулятор,уравнения с модулем калькулятор онлайн,уравнения с модулем онлайн,уравнения с модулем онлайн калькулятор,уравнения с модулем примеры,уравнения с модулем примеры решения,уравнения с модулем простейшие,уравнения с модулем решение,уравнения с модулем решение онлайн,уравнения с модулем решить онлайн,уравнения с модулем с двойным модулем,уравнения с модулем сложные,уравнения с модулями,уравнения с модулями 10 класс,уравнения с модулями 7 класс,уравнения с модулями 7 класс в ответе 0,уравнения с модулями 7 класс объяснение,уравнения с модулями как решать,уравнения с модулями примеры решений,уравнения содержащие модуль,х 1 модуль,х 2 модуль,х 2 модуль 3,х 3 2 модуль,х 5 модуль,х модуль. На этой странице вы найдёте калькулятор, который поможет решить любой вопрос, в том числе и 1 x модуль. Просто введите задачу в окошко и нажмите «решить» здесь (например, 1 модуль x 2).

Где можно решить любую задачу по математике, а так же 1 x модуль Онлайн?

Решить задачу 1 x модуль вы можете на нашем сайте https://pocketteacher.ru. Бесплатный онлайн решатель позволит решить онлайн задачу любой сложности за считанные секунды. Все, что вам необходимо сделать — это просто ввести свои данные в решателе. Так же вы можете посмотреть видео инструкцию и узнать, как правильно ввести вашу задачу на нашем сайте. А если у вас остались вопросы, то вы можете задать их в чате снизу слева на странице калькулятора.

Двойной модуль как решать

Tel. +380685083397
[email protected]
skype, facebook:
roman.yukhym

Решение задач
Андрей

facebook:
dniprovets25

Как решить простейшее модульное уравнение или уравнение содержащее модуль?

Обычно решение сводится к системе :
Уравнения содержащие модуль

Сразу рассмотрим на примере решение уравнений.

Решите уравнение | x – 6| = 18.

Выражение стоящее под модулем приравниваем к 0:

Отмечаем 6 на координатной прямой, далее проверяем знак на каждом из получившихся интервалах.

На интервале (-∞; 6) возьмем число 0 и подставим
0-6=-6 получилось отрицательное число, значит на этом интервале будет знак “ – ”

На интервале (6;+∞) возьмем число 7 и подставим
7-6=1 получилось положительное число, значит на этом интервале будет знак “ + ”

Числовая прямая

Теперь решаем уравнения на каждом интервале.

(-∞; 6) здесь получился знак “ – ”, значит выражение под модулем поменяет знаки на противоположные:

Видно, что -12 лежит на интервале (-∞; 6) следовательно, является корнем уравнения.

(6;+∞) здесь получился знак “ + ”, значит выражение под модулем остается без изменения:

Видно, что 24 лежит на интервале (6;+∞) следовательно, является корнем уравнения.

Решите уравнение | 2x – 5 |- | 4 — x | = -18.

Выражения стоящие под модулем приравниваем к 0:

2x – 5 = 0 и 4 — x = 0
x=2,5 и x=4

Отмечаем x=2,5 и x=4 на координатной прямой, далее проверяем знак на каждом из получившихся интервалах.

На интервале (-∞; 2,5) возьмем число 0 и подставим в каждое выражение
2*0-5=-5 получилось отрицательное число, значит на этом интервале будет знак “ – ”
4-0=4 получилось положительное число, значит на этом интервале будет знак “ + ”

На интервале (2,5; 4) возьмем число 3 и подставим в каждое выражение
2*3-5=1 получилось положительное число, значит на этом интервале будет знак “ + ”
4-3=1 получилось положительное число, значит на этом интервале будет знак “ + ”

На интервале (4; +∞) возьмем число 5 и подставим в каждое выражение
2*5-5=5 получилось положительное число, значит на этом интервале будет знак “ + ”
4-5=-1 получилось отрицательное число, значит на этом интервале будет знак “ – ”

Теперь решаем уравнения на каждом интервале.

(-∞; 2,5) здесь получился знак “ – ” у выражения “ 2x – 5 ”, значит выражение под модулем поменяет знаки на противоположные и знак “ + ” у выражения “ 4 — x ”, значит выражение под модулем остается без изменения:

-2x + 5 — ( 4 — x ) = -18
-2x + 5 — 4 + x = -18
x=19
Видно, что 19 не лежит на интервале (-∞; 2,5) следовательно, не является корнем уравнения.

(2,5; 4) здесь получился знак “ + ” у обоих выражений, значит выражения под модулем останутся без изменений:

2x – 5 — ( 4 — x ) = -18
2x – 5 — 4 + x = -18
3x=-9
x=-3

Видно, что -3 лежит на интервале (2,5; 4) следовательно,не является корнем уравнения.

(4; +∞) здесь получился знак “ – ” у выражения “ 4 — x ”, значит выражение под модулем поменяет знаки на противоположные и знак “ + ” у выражения “ 2x – 5 ”, значит выражение под модулем остается без изменения:

2x – 5 — ( — 4 + x ) = -18
2x – 5 + 4 — x = -18
x=-17

Видно, что -17 лежит на интервале (4; +∞) следовательно,не является корнем уравнения.

Ответ: корней нет

Решите уравнение ||x|-3|=15.

Так как в правой части стоит простое число то распишем на два уравнения (раскроем внешний модуль):

Перенесем в обоих уравнениях -3 вправо, получим:

|x|=18
|x|=-12 (модуль не может равняться отрицательному числу, следовательно это уравнение не имеет решений)

Раскрываем модуль |x|=18

Хочешь готовиться к экзаменам бесплатно? Репетитор онлайн бесплатно. Без шуток. ЗДЕСЬ

Хочешь подготовиться к ОГЭ или ЕГЭ по математике на отлично?

Хочешь проверить свои силы и узнать результат насколько ты готов к ЕГЭ или ОГЭ?

Важное замечание!
Если вместо формул ты видишь абракадабру, почисти кэш. Как это сделать в твоем браузере написано здесь: «Как почистить кэш браузера».

Надеюсь, ты уже усвоил тему «Модуль числа» и таким образом уже частично готов к ЕГЭ по математике? 🙂

Если нет, срочно повтори эту тему. А если да, читай дальше.

Решение уравнений с модулем может быть самостоятельной задачей, но часто такие уравнения могут возникнуть при решении уравнений другого типа, например, квадратных.

Вот пример подобной ситуации:

Видно, что в правой части – квадрат числа :

Казалось бы, теперь достаточно просто убрать квадраты слева и справа, и получим линейное уравнение.

В таких ситуациях нужно быть предельно осторожным: ведь ты же помнишь простое правило:

Вот и появляется на сцене наш модуль:

Чтобы не теряться в таких случаях, давай разберемся, что из себя представляет решение уравнений с модулем.

Let’s dive right in. (Поехали!)

Решение уравнений с модулем вида |Х| = a

Уравнения такого вида решаем, основываясь на свойствах модуля, которые мы разобрали в теме «Модуль числа» .

Давай разбираться на примерах. Необходимо решить уравнение вида:

Это просто , если больше либо равно нулю, или , если меньше нуля.

То есть можно формализовано записать так:

А если вот такое уравнение:

Эти рассуждения можно было и обойти, вспомнив основное свойство модуля:

Модуль всегда положителен либо равен нулю!

Если обобщить разобранные выше примеры, то можно написать общее правило для решения уравнений вида :

Попробуем применить это правило для такого уравнения:

Выражение под знаком модуля изменилось, но на логике рассуждений это не отражается, поэтому давай решать уравнение, применяя наше правило:

В нашем примере под » » подразумевается » «, а значение . Зная это, получаем:

А если уравнение имеет вид:

Что-то меняется в рассуждениях? Конечно, нет! Ну, тогда давай решать его!

Уловил? Закрепим на примерах.

Примеры для самостоятельной работы

Решения примеров для самостоятельной работы

Точно так же как и в предыдущем примере уравнения с модулем могут возникнуть при решении уравнений другого типа, например, иррациональных.

Вот пример подобной ситуации:

Мы могли бы раскрыть скобки, перенести все в одну сторону, привести подобные и решить обычное квадратное уравнение (например, через дискриминант).

Но здесь удобнее поступить по-другому!

Заметим, что в правой части уравнения – формула сокращённого умножения квадрат суммы:

Тогда уравнение станет таким:

Казалось бы, теперь достаточно просто убрать квадраты слева и справа, и получим линейное уравнение.

Будь предельно осторожным: опять вспоминаем простое правило: ?

И опять на сцене наш модуль:

Чтобы не теряться в таких случаях, научимся решать уравнения с модулем (все три типа).

Три типа уравнений с модулем

1. Уравнения вида |X| = |a|

Большинство уравнений с модулем можно решить, используя одно только определение модуля. Например:

Решите уравнение:

Это просто , если , или , если .

Ответ:

Другой пример:

И правда, вспомним свойство №1:

, то есть модуль всегда неотрицателен.

Итак, мы выработали общее правило решения простейших уравнений с модулем:

Ещё примеры (как обычно, пробуй решить их сам, потом смотри решения):

Решения:​

2. Уравнения вида |X| = |Y|

Если начнём раскрывать модули по определению, натолкнёмся на множество проверок: какое число больше нуля, какое меньше; в итоге получим большую совокупность, которая затем упростится.

Но можно сделать так, чтобы сразу было всё кратко.

Для этого вспомним свойство модуля №7:

С помощью этого свойства можем избавляться от модулей:

Пример:

Решение:

Реши самостоятельно:

Ответы:

3. Уравнения вида |X| = Y

Отличие от первого типа уравнений в том, что в правой части тоже переменная. А она может быть как положительной, так и отрицательной.

Поэтому в её неотрицательности нужно специально убедиться, ведь модуль не может равняться отрицательному числу (свойство №1):

Пример:

Решение:

Если пропустить проверку на неотрицательность правой части, можно ошибочно написать в ответе сторонние корни, и таким образом потерять баллы. Давайте проверим: действительно ли надо выбросить корень ? Подставим его в исходное уравнение :

Теперь задачи для самостоятельного решения:

Ответы:

Решим квадратные уравнения и . Дискриминант у них одинаковый:

Итак, исходное уравнение равносильно системе

Ответ:

Метод интервалов в задачах с модулем

Пример:

Решение:

Рассмотрим первый модуль . По определению он раскрывается «с плюсом» (то есть выражение под модулем не меняется), если , и «с минусом» (то есть все знаки меняются на противоположные), если :

Аналогично и со вторым:

Проблема только в том, что теперь нам нужно рассмотреть очень много вариантов: по варианта для каждого модуля, итого четыре разных, но похожих друг на друга, уравнения.

Если модулей будет не два, а три, получится уже уравнений!

Можно ли как-то сократить количество вариантов?

Да, можно – ведь не все условия могут выполняться одновременно: и противоречат друг другу.

Поэтому нет смысла раскрывать второй модуль «с плюсом», если первый раскрыт «с минусом». Значит, здесь у нас на одно уравнение меньше.

Теперь систематизируем то, что мы только что выяснили, и разработаем последовательность действий в таких примерах:

1. Определим корни подмодульных выражений – такие , при которых выражения равны нулю:

2. Отметим корни выражений под модулями на числовой оси:

3. Подпишем у каждого из получившихся интервалов, какой знак принимает каждое из наших подмодульных выражений.

4. Для каждого интервала запишем и решим уравнение. Важно проследить, чтобы ответы соответствовали интервалу!

I. . Здесь оба модуля раскрываем «с минусом»:

-3″> – этот корень сторонний.

II. . Здесь первый модуль раскрываем «с плюсом», а второй – «с минусом»:

– этот корень попадает в «свой» интервал, значит, он подходит.

III. . Здесь оба модуля раскрываем «с плюсом»:

– этот корень тоже является решением.

Проверим полученные корни:

I. (корень и правда сторонний).

II. .

III. .

Ответ:

Примеры:

Решения:

Модуль в модуле

В некоторых уравнениях встречается «вложенный» модуль, то есть модуль какого-то выражения является частью подмодульного выражения, например:

Что делать в таком случае? Все банально: раскрывать модули. Но раскрывать их нужно по очереди. Какой будем раскрывать первым?

А это зависит от того, каким методом ты хочешь решить это уравнение. Рассмотрим два возможных варианта:

I. Данное уравнение является уравнением вида

В этом случае первый способ решения будет стандартным для такого типа:

– подмодульное выражение – в нашем примере это , то есть:

Получили два элементарных уравнения такого же типа, то есть:

Эти четыре числа и будут ответом, можешь проверить их подстановкой в исходное уравнение.

II. Есть ещё один, более универсальный способ, который подойдёт для любых задач, не попадающих ни в какой из стандартных типов.

Что это за метод?

Метод интервалов.

В этом случае нужно раскрывать модули начиная с самых «глубоких», то есть «внутренних». В нашем случае внутренним будет модуль, выделенный красным цветом:

Чтобы раскрыть его, надо рассмотреть 2 случая: и , то есть уравнение распадается на два уравнения:

Краткое изложение статьи и основные формулы

Уравнения с модулем делятся на три вида, каждый вид имеет свой подход к решению:

1. Уравнения вида

2. Уравнения вида .

3. Уравнения вида .

Теперь тебе слово.

Как тебе. про уравнения с модулем? Легкотня! )

Напиши внизу в комментариях помогла тебе наша статья или нет.

Расскажи о своем опыте решения уравнений с модулем, если он у тебя был.

Возможно у тебя есть вопросы. Или предложения.

Напиши в комментариях. Мы читаем все.

И удачи на экзаменах!

P.S. ПОСЛЕДНИЙ БЕСЦЕННЫЙ СОВЕТ 🙂

Ну вот, тема закончена. Если ты читаешь эти строки, значит ты очень крут.

Потому что только 5% людей способны освоить что-то самостоятельно. И если ты дочитал до конца, значит ты попал в эти 5%!

Теперь самое главное.

Ты разобрался с теорией по этой теме. И, повторюсь, это… это просто супер! Ты уже лучше, чем абсолютное большинство твоих сверстников.

Проблема в том, что этого может не хватить…

Для успешной сдачи ОГЭ или ЕГЭ, для перехода в 10-й класс или поступления в институт на бюджет и, САМОЕ ГЛАВНОЕ, для жизни.

Я не буду тебя ни в чем убеждать, просто скажу одну вещь…

Люди, получившие хорошее образование, зарабатывают намного больше, чем те, кто его не получил. Это статистика.

Но и это – не главное.

Главное то, что они БОЛЕЕ СЧАСТЛИВЫ (есть такие исследования). Возможно потому, что перед ними открывается гораздо больше возможностей и жизнь становится ярче? Не знаю.

Что нужно, чтобы быть наверняка лучше других на ЕГЭ и быть в конечном итоге… более счастливым?

НАБИТЬ РУКУ, РЕШАЯ ЗАДАЧИ ПО ЭТОЙ ТЕМЕ.

На экзамене у тебя не будут спрашивать теорию.

Тебе нужно будет решать задачи на время.

И, если ты не решал их (МНОГО!), ты обязательно где-нибудь глупо ошибешься или просто не успеешь.

Это как в спорте – нужно много раз повторить, чтобы выиграть наверняка.

Найди где хочешь сборник, обязательно с решениями, подробным разбором и решай, решай, решай!

Я рекомендую использовать для этих целей наш учебник «YouClever» (который ты сейчас читаешь) и решебник и программу подготовки «100gia».

Условия их приобретения изложены здесь. Кликните по этой ссылке, приобретите доступ к YouClever и 100gia и начните готовиться прямо сейчас!

И в заключение.

Если наши задачи тебе не нравятся, найди другие. Только не останавливайся на теории.

“Понял” и “Умею решать” – это совершенно разные навыки. Тебе нужны оба.

Найди задачи и решай!

Комментарии

Спасибо огромное,повторил,сдал на отлично,Алексею нобелевскую по математике)

Марк, наши поздравления с отличной сдачей. Премию Алексею передам 🙂

нобелевские по математике не присуждаются .

Наградим поощрительной грамотой )

Добрый день! В пункте №3 Уравнения вида ∣x∣=y во втором примере: −2∣x+4∣=3−x, откуда дальше в решении появляется коэффициент 4 в правой части? −2∣x+4∣=3−4x Спасибо за ответ и Ваш чудесный и полезный сайт!

Роман, привет! Спасибо за замечания и слова благодарности. Очень ценно. Алексей Шевчук проверит и поправит, если там ошибка. Еще раз спасибо!

Роман, спасибо. Это была опечатка в условии.

А как решить такой пример 7|2-4|+4*-8

помогите,пожалуйста,решить уравнение дробь в модуле :числитель 13,296 знаменатель 3.71 минус модуль 0,4х минус4,7 модуль закрывается,далее от дроби минус 2,2 умножить на 1,4.Еще раз обращаю внимание: сама дробь в модуле И равно 8 Пожалуйста помогите

Здравствуйте, помогите пожалуйста решить такое уравнение |x-1|=2x+3

Спасибо большое . Сайт замечательный ,я смогла разобраться и понять материал . Создателям огромное спасибо ,их работа заслуживает высокого внимания . Перейду на родной язык: Danke schön. Ihre Arbeit ist wirklich wunderschön. Danke ein male.

Gern geschehen, Dascha! Bitte. International Mathematical Unterstützung zu Ihren Diensten ))

как по графику кусочно заданной функции записать уравнение, содержащее несколько модулей вида y=a|x|+b|x-8|+x+c? №23 ОГЭ систему составила y= -2x-4 . x 8 , а как перейти к другой записи уравнения

Очень хорошо разобрано и объяснено. И за советы спасибо)

Лера, жму руку! Спасибо за теплые слова. Удачи на всех экзаменах!

Решите уравнение ∣x∣=−3. разве может модуль равняться отрицательному числу

Джозеф, нет, не может, и в этом примере поясняется, почему.

Помогите решить |х|+|y-x|=2 Нужно расскрыть модуль и по получившимся ответам которых 4 как сказал препод

Виталий, в самом начале раздела «Метод интервалов в задачах с модулем» показано, как раскрывать сумму модулей. Принципиально это ничем не отличается от раскрытия одного модуля, просто будет больше комбинаций – 4 штуки, по 2 на каждый модуль: 1) x>=0, y>=x; 2) x =x; 3) . и так далее

Как раскрыть модуль в модуле в уравнении. Уравнения с модулем. Решение неравенств с модулем

Инструкция

Если модуль представлен в виде непрерывной функции, то значение ее аргумента может быть как положительным, так и отрицательным: |х| = х, х ≥ 0; |х| = — х, х

Модуль нулю, а модуль любого положительного числа – ему . Если аргумент отрицательный, то после раскрытия скобок его знак меняется с минуса на плюс. На основании этого вытекает вывод, что модули противоположных равны: |-х| = |х| = х.

Модуль комплексного числа находится по формуле: |a| = √b ² + c ², а |a + b| ≤ |a| + |b|. Если в аргументе присутствует в виде множителя положительное число, то его можно вынести за знак скобки, например: |4*b| = 4*|b|.

Если аргумент представлен в виде сложного числа, то для удобства вычислений допускается порядка членов выражения, заключенного в прямоугольные скобки: |2-3| = |3-2| = 3-2 = 1, поскольку (2-3) меньше нуля.

Возведенный в степень аргумент одновременно находится под знаком корня того же порядка – он решается при помощи : √a² = |a| = ±a.

Если перед вами задача, в которой не указано условие раскрытия скобок модуля, то избавляться от них не нужно – это и будет конечный результат. А если требуется их раскрыть, то необходимо указать знак ±. Например, нужно найти значение выражения √(2 * (4-b)) ². Его решение выглядит следующим образом: √(2 * (4-b)) ² = |2 * (4-b)| = 2 * |4-b|. Поскольку знак выражения 4-b неизвестен, то его нужно оставить в скобках. Если добавить дополнительное условие, например, |4-b| >

Модуль нуля равен нулю, а модуль любого положительного числа – ему самому. Если аргумент отрицательный, то после раскрытия скобок его знак меняется с минуса на плюс. На основании этого вытекает вывод, что модули противоположных чисел равны: |-х| = |х| = х.

Модуль комплексного числа находится по формуле: |a| = √b ² + c ², а |a + b| ≤ |a| + |b|. Если в аргументе присутствует в виде множителя целое положительное число, то его можно вынести за знак скобки, например: |4*b| = 4*|b|.

Отрицательным модуль быть не может, поэтому любое отрицательное число преобразуется в положительное: |-x| = x, |-2| = 2, |-1/7| = 1/7, |-2,5| = 2,5.

Если аргумент представлен в виде сложного числа, то для удобства вычислений допускается изменение порядка членов выражения, заключенного в прямоугольные скобки: |2-3| = |3-2| = 3-2 = 1, поскольку (2-3) меньше нуля.

Если перед вами задача, в которой не указано условие раскрытия скобок модуля, то избавляться от них не нужно – это и будет конечный результат. А если требуется их раскрыть, то необходимо указать знак ±. Например, нужно найти значение выражения √(2 * (4-b)) ². Его решение выглядит следующим образом: √(2 * (4-b)) ² = |2 * (4-b)| = 2 * |4-b|. Поскольку знак выражения 4-b неизвестен, то его нужно оставить в скобках. Если добавить дополнительное условие, например, |4-b| > 0, то в итоге получится 2 * |4-b| = 2 *(4 — b). В качестве неизвестного элемента также может быть задано конкретное число, которое следует принимать во внимание, т.к. оно будет влиять на знак выражения.

Модуль – это абсолютная величина выражения. Чтобы хоть как-то обозначить модуль, принято использовать прямые скобки. То значение, которое заключено в ровных скобках, и является тем значением, которое взято по модулю. Процесс решения любого модуля заключается в раскрытии тех самых прямых скобок, которые математическим языком именуются модульными скобками. Их раскрытие происходит по определенному ряду правил. Также, в порядке решения модулей, находятся и множества значений тех выражений, которые находились в модульных скобках. В большей части всех случаев, модуль раскрывается таким способом, что выражение, которое было подмодульным, получает и положительные, и отрицательные значения, в числе которых также и значение ноль. Если отталкиваться от установленных свойств модуля, то в процессе составляются различные уравнения или же неравенства от исходного выражения, которые затем необходимо решить. Разберемся же с тем, как решать модули.

Процесс решения

Решение модуля начинается с записи исходного уравнения с модулем. Чтобы ответить на вопрос о том, как решать уравнения с модулем, нужно раскрыть его полностью. Для решения такого уравнения, модуль раскрывается. Все модульные выражения должны быть рассмотрены. Следует определить при каких значениях неизвестных величин, входящих в его состав, модульное выражение в скобках обращается в ноль. Для того чтобы это сделать, достаточно приравнять выражение в модульных скобках к нулю, а затем высчитать решение образовавшегося уравнения. Найденные значения нужно зафиксировать. Таким же способом нужно определить еще и значение всех неизвестных переменных для всех модулей в данном уравнении. Далее необходимо заняться определением и рассмотрением всех случаев существования переменных в выражениях, когда они отличны от значения ноль. Для этого нужно записать некоторую систему из неравенств соответственно всем модулям в исходном неравенстве. Неравенства должны быть составлены так, чтоб они охватывали все имеющиеся и возможные значения для переменной, которые находят на числовой прямой. Затем нужно начертить для визуализации эту самую числовую прямую, на которой в дальнейшем отложить все полученные значения.

Практически все сейчас можно сделать в интернете. Не является исключением из правил и модуль. Решить онлайн его можно на одном из многочисленных современных ресурсов. Все те значения переменной, которые находятся в нулевом модуле, будут особым ограничением, которое будет использовано в процессе решения модульного уравнения. В исходном уравнении требуется раскрыть все имеющиеся модульные скобки, при этом, изменяя знак выражения, таким образом, чтобы значения искомой переменной совпадали с теми значениями, которые видно на числовой прямой. Полученное уравнение необходимо решить. То значение переменной, которое будет получено в ходе решения уравнения, нужно проверять на ограничение, которое задано самим модулем. Если значение переменной полностью удовлетворяет условие, то оно является правильным. Все корни, которые будут получены в ходе решения уравнения, но не будут подходить по ограничениям, должны быть отброшены.

Одна из самых сложных тем для учащихся – это решение уравнений, содержащих переменную под знаком модуля. Давайте разберемся для начала с чем же это связано? Почему, например, квадратные уравнения большинство детей щелкает как орешки, а с таким далеко не самым сложным понятием как модуль имеет столько проблем?

На мой взгляд, все эти сложности связаны с отсутствием четко сформулированных правил для решения уравнений с модулем. Так, решая квадратное уравнение, ученик точно знает, что ему нужно сначала применять формулу дискриминанта, а затем формулы корней квадратного уравнения. А что делать, если в уравнении встретился модуль? Постараемся четко описать необходимый план действий на случай, когда уравнение содержит неизвестную под знаком модуля. К каждому случаю приведем несколько примеров.

Но для начала вспомним определение модуля . Итак, модулем числа a называется само это число, если a неотрицательно и -a , если число a меньше нуля. Записать это можно так:

|a| = a, если a ≥ 0 и |a| = -a, если a

Говоря о геометрическом смысле модуля, следует помнить, что каждому действительному числу соответствует определенная точка на числовой оси – ее координата. Так вот, модулем или абсолютной величиной числа называется расстояние от этой точки до начала отсчета числовой оси. Расстояние всегда задается положительным числом. Таким образом, модуль любого отрицательного числа есть число положительное. Кстати, даже на этом этапе многие ученики начинают путаться. В модуле может стоять какое угодно число, а вот результат применения модуля всегда число положительное.

Теперь перейдем непосредственно к решению уравнений.

1. Рассмотрим уравнение вида |x| = с, где с – действительное число. Это уравнение можно решить с помощью определения модуля.

Все действительные числа разобьем на три группы: те, что больше нуля, те, что меньше нуля, и третья группа – это число 0. Запишем решение в виде схемы:

{±c, если с > 0

Если |x| = c, то x = {0, если с = 0

{нет корней, если с

1) |x| = 5, т.к. 5 > 0, то x = ±5;

2) |x| = -5, т.к. -5

3) |x| = 0, то x = 0.

2. Уравнение вида |f(x)| = b, где b > 0. Для решения данного уравнения необходимо избавиться от модуля. Делаем это так: f(x) = b или f(x) = -b. Теперь необходимо решить отдельно каждое из полученных уравнений. Если в исходном уравнении b

1) |x + 2| = 4, т.к. 4 > 0, то

x + 2 = 4 или x + 2 = -4

2) |x 2 – 5| = 11, т.к. 11 > 0, то

x 2 – 5 = 11 или x 2 – 5 = -11

x 2 = 16 x 2 = -6

x = ± 4 нет корней

3) |x 2 – 5x| = -8 , т.к. -8

3. Уравнение вида |f(x)| = g(x). По смыслу модуля такое уравнение будет иметь решения, если его правая часть больше или равна нулю, т.е. g(x) ≥ 0. Тогда будем иметь:

f(x) = g(x) или f(x) = -g(x) .

1) |2x – 1| = 5x – 10. Данное уравнение будет иметь корни, если 5x – 10 ≥ 0. Именно с этого и начинают решение таких уравнений.

1. О.Д.З. 5x – 10 ≥ 0

2. Решение:

2x – 1 = 5x – 10 или 2x – 1 = -(5x – 10)

3. Объединяем О.Д.З. и решение, получаем:

Корень x = 11/7 не подходит по О.Д.З., он меньше 2, а x = 3 этому условию удовлетворяет.

Ответ: x = 3

2) |x – 1| = 1 – x 2 .

1. О.Д.З. 1 – x 2 ≥ 0. Решим методом интервалов данное неравенство:

(1 – x)(1 + x) ≥ 0

2. Решение:

x – 1 = 1 – x 2 или x – 1 = -(1 – x 2)

x 2 + x – 2 = 0 x 2 – x = 0

x = -2 или x = 1 x = 0 или x = 1

3. Объединяем решение и О.Д.З.:

Подходят только корни x = 1 и x = 0.

Ответ: x = 0, x = 1.

4. Уравнение вида |f(x)| = |g(x)|. Такое уравнение равносильно двум следующим уравнениям f(x) = g(x) или f(x) = -g(x).

1) |x 2 – 5x + 7| = |2x – 5|. Данное уравнение равносильно двум следующим:

x 2 – 5x + 7 = 2x – 5 или x 2 – 5x +7 = -2x + 5

x 2 – 7x + 12 = 0 x 2 – 3x + 2 = 0

x = 3 или x = 4 x = 2 или x = 1

Ответ: x = 1, x = 2, x = 3, x = 4.

5. Уравнения, решаемые методом подстановки (замены переменной). Данный метод решения проще всего объяснить на конкретном примере. Так, пусть дано квадратное уравнение с модулем:

x 2 – 6|x| + 5 = 0. По свойству модуля x 2 = |x| 2 , поэтому уравнение можно переписать так:

|x| 2 – 6|x| + 5 = 0. Сделаем замену |x| = t ≥ 0, тогда будем иметь:

t 2 – 6t + 5 = 0. Решая данное уравнение, получаем, что t = 1 или t = 5. Вернемся к замене:

|x| = 1 или |x| = 5

x = ±1 x = ± 5

Ответ: x = -5, x = -1, x = 1, x = 5.

Рассмотрим еще один пример:

x 2 + |x| – 2 = 0. По свойству модуля x 2 = |x| 2 , поэтому

|x| 2 + |x| – 2 = 0. Сделаем замену |x| = t ≥ 0, тогда:

t 2 + t – 2 = 0. Решая данное уравнение, получаем, t = -2 или t = 1. Вернемся к замене:

|x| = -2 или |x| = 1

Нет корней x = ± 1

Ответ: x = -1, x = 1.

6. Еще один вид уравнений – уравнения со «сложным» модулем. К таким уравнениям относятся уравнения, в которых есть «модули в модуле». Уравнения данного вида можно решать, применяя свойства модуля.

1) |3 – |x|| = 4. Будем действовать так же, как и в уравнениях второго типа. Т.к. 4 > 0, то получим два уравнения:

3 – |x| = 4 или 3 – |x| = -4.

Теперь выразим в каждом уравнении модуль х, тогда |x| = -1 или |x| = 7.

Решаем каждое из полученных уравнений. В первом уравнении нет корней, т.к. -1

Ответ x = -7, x = 7.

2) |3 + |x + 1|| = 5. Решаем это уравнение аналогичным образом:

3 + |x + 1| = 5 или 3 + |x + 1| = -5

|x + 1| = 2 |x + 1| = -8

x + 1 = 2 или x + 1 = -2. Нет корней.

Ответ: x = -3, x = 1.

Существует еще и универсальный метод решения уравнений с модулем. Это метод интервалов. Но мы его рассмотрим в дальнейшем.

сайт, при полном или частичном копировании материала ссылка на первоисточник обязательна.

Одна из самых сложных тем для учащихся – это решение уравнений, содержащих переменную под знаком модуля. Давайте разберемся для начала с чем же это связано? Почему, например, квадратные уравнения большинство детей щелкает как орешки, а с таким далеко не самым сложным понятием как модуль имеет столько проблем?

На мой взгляд, все эти сложности связаны с отсутствием четко сформулированных правил для решения уравнений с модулем. Так, решая квадратное уравнение, ученик точно знает, что ему нужно сначала применять формулу дискриминанта, а затем формулы корней квадратного уравнения. А что делать, если в уравнении встретился модуль? Постараемся четко описать необходимый план действий на случай, когда уравнение содержит неизвестную под знаком модуля. К каждому случаю приведем несколько примеров.

Но для начала вспомним определение модуля . Итак, модулем числа a называется само это число, если a неотрицательно и -a , если число a меньше нуля. Записать это можно так:

|a| = a, если a ≥ 0 и |a| = -a, если a

Говоря о геометрическом смысле модуля, следует помнить, что каждому действительному числу соответствует определенная точка на числовой оси – ее координата. Так вот, модулем или абсолютной величиной числа называется расстояние от этой точки до начала отсчета числовой оси. Расстояние всегда задается положительным числом. Таким образом, модуль любого отрицательного числа есть число положительное. Кстати, даже на этом этапе многие ученики начинают путаться. В модуле может стоять какое угодно число, а вот результат применения модуля всегда число положительное.

Теперь перейдем непосредственно к решению уравнений.

1. Рассмотрим уравнение вида |x| = с, где с – действительное число. Это уравнение можно решить с помощью определения модуля.

Все действительные числа разобьем на три группы: те, что больше нуля, те, что меньше нуля, и третья группа – это число 0. Запишем решение в виде схемы:

{±c, если с > 0

Если |x| = c, то x = {0, если с = 0

{нет корней, если с

1) |x| = 5, т.к. 5 > 0, то x = ±5;

2) |x| = -5, т.к. -5

3) |x| = 0, то x = 0.

2. Уравнение вида |f(x)| = b, где b > 0. Для решения данного уравнения необходимо избавиться от модуля. Делаем это так: f(x) = b или f(x) = -b. Теперь необходимо решить отдельно каждое из полученных уравнений. Если в исходном уравнении b

1) |x + 2| = 4, т.к. 4 > 0, то

x + 2 = 4 или x + 2 = -4

2) |x 2 – 5| = 11, т.к. 11 > 0, то

x 2 – 5 = 11 или x 2 – 5 = -11

x 2 = 16 x 2 = -6

x = ± 4 нет корней

3) |x 2 – 5x| = -8 , т.к. -8

3. Уравнение вида |f(x)| = g(x). По смыслу модуля такое уравнение будет иметь решения, если его правая часть больше или равна нулю, т.е. g(x) ≥ 0. Тогда будем иметь:

f(x) = g(x) или f(x) = -g(x) .

1) |2x – 1| = 5x – 10. Данное уравнение будет иметь корни, если 5x – 10 ≥ 0. Именно с этого и начинают решение таких уравнений.

1. О.Д.З. 5x – 10 ≥ 0

2. Решение:

2x – 1 = 5x – 10 или 2x – 1 = -(5x – 10)

3. Объединяем О.Д.З. и решение, получаем:

Корень x = 11/7 не подходит по О.Д.З., он меньше 2, а x = 3 этому условию удовлетворяет.

Ответ: x = 3

2) |x – 1| = 1 – x 2 .

1. О.Д.З. 1 – x 2 ≥ 0. Решим методом интервалов данное неравенство:

(1 – x)(1 + x) ≥ 0

2. Решение:

x – 1 = 1 – x 2 или x – 1 = -(1 – x 2)

x 2 + x – 2 = 0 x 2 – x = 0

x = -2 или x = 1 x = 0 или x = 1

3. Объединяем решение и О.Д.З.:

Подходят только корни x = 1 и x = 0.

Ответ: x = 0, x = 1.

4. Уравнение вида |f(x)| = |g(x)|. Такое уравнение равносильно двум следующим уравнениям f(x) = g(x) или f(x) = -g(x).

1) |x 2 – 5x + 7| = |2x – 5|. Данное уравнение равносильно двум следующим:

x 2 – 5x + 7 = 2x – 5 или x 2 – 5x +7 = -2x + 5

x 2 – 7x + 12 = 0 x 2 – 3x + 2 = 0

x = 3 или x = 4 x = 2 или x = 1

Ответ: x = 1, x = 2, x = 3, x = 4.

5. Уравнения, решаемые методом подстановки (замены переменной). Данный метод решения проще всего объяснить на конкретном примере. Так, пусть дано квадратное уравнение с модулем:

x 2 – 6|x| + 5 = 0. По свойству модуля x 2 = |x| 2 , поэтому уравнение можно переписать так:

|x| 2 – 6|x| + 5 = 0. Сделаем замену |x| = t ≥ 0, тогда будем иметь:

t 2 – 6t + 5 = 0. Решая данное уравнение, получаем, что t = 1 или t = 5. Вернемся к замене:

|x| = 1 или |x| = 5

x = ±1 x = ± 5

Ответ: x = -5, x = -1, x = 1, x = 5.

Рассмотрим еще один пример:

x 2 + |x| – 2 = 0. По свойству модуля x 2 = |x| 2 , поэтому

|x| 2 + |x| – 2 = 0. Сделаем замену |x| = t ≥ 0, тогда:

t 2 + t – 2 = 0. Решая данное уравнение, получаем, t = -2 или t = 1. Вернемся к замене:

|x| = -2 или |x| = 1

Нет корней x = ± 1

Ответ: x = -1, x = 1.

6. Еще один вид уравнений – уравнения со «сложным» модулем. К таким уравнениям относятся уравнения, в которых есть «модули в модуле». Уравнения данного вида можно решать, применяя свойства модуля.

1) |3 – |x|| = 4. Будем действовать так же, как и в уравнениях второго типа. Т.к. 4 > 0, то получим два уравнения:

3 – |x| = 4 или 3 – |x| = -4.

Теперь выразим в каждом уравнении модуль х, тогда |x| = -1 или |x| = 7.

Решаем каждое из полученных уравнений. В первом уравнении нет корней, т.к. -1

Ответ x = -7, x = 7.

2) |3 + |x + 1|| = 5. Решаем это уравнение аналогичным образом:

3 + |x + 1| = 5 или 3 + |x + 1| = -5

|x + 1| = 2 |x + 1| = -8

x + 1 = 2 или x + 1 = -2. Нет корней.

Ответ: x = -3, x = 1.

Существует еще и универсальный метод решения уравнений с модулем. Это метод интервалов. Но мы его рассмотрим в дальнейшем.

blog.сайт, при полном или частичном копировании материала ссылка на первоисточник обязательна.

Термин (module) в буквальном переводе с латинского означает «мера». Это понятие было введено в математику английским учёным Р. Котесом. А немецкий математик К. Вейерштрасс ввёл в обращение знак модуля — символ, которым это понятие обозначается при написании.

Вконтакте

Впервые данное понятие изучается в математике по программе 6 класса средней школы. Согласно одному из определений, модуль — это абсолютное значение действительного числа. Другими словами, чтобы узнать модуль действительного числа, необходимо отбросить его знак.

Графически абсолютное значение а обозначается как |a| .

Основная отличительная черта этого понятия заключается в том, что он всегда является неотрицательной величиной.

Числа, которые отличаются друг от друга только знаком, называются противоположными. Если значение положительное, то противоположное ему будет отрицательным, а ноль является противоположным самому себе.

Геометрическое значение

Если рассматривать понятие модуля с позиций геометрии, то он будет обозначать расстояние, которое измеряется в единичных отрезках от начала координат до заданной точки. Это определение полностью раскрывает геометрический смысл изучаемого термина.

Графически это можно выразить следующим образом: |a| = OA.

Свойства абсолютной величины

Ниже будут рассмотрены все математические свойства этого понятия и способы записи в виде буквенных выражений:

Особенности решения уравнений с модулем

Если говорить о решении математических уравнений и неравенств, в которых содержится module, то необходимо помнить, что для их решения потребуется открыть этот знак.

К примеру, если знак абсолютной величины содержит в себе некоторое математическое выражение, то перед тем как раскрыть модуль, необходимо учитывать действующие математические определения.

|А + 5| = А + 5 , если, А больше или равняется нулю.

5-А , если, А значение меньше нуля.

В некоторых случаях знак может раскрываться однозначно при любых значениях переменной.

Рассмотрим ещё одни пример. Построим координатную прямую, на которой отметим все числовые значения абсолютной величиной которых будет 5.

Для начала необходимо начертить координатную прямую, обозначить на ней начало координат и задать размер единичного отрезка. Кроме того, прямая должна иметь направление. Теперь на этой прямой необходимо нанести разметки, которые будут равны величине единичного отрезка.

Таким образом, мы можем увидеть, что на этой координатной прямой будут две интересующие нас точки со значениями 5 и -5.

Уравнения и неравенства с модулем

1. Уравнения и неравенства с модулем часть 2

2. Уравнение вида | f(x)| = g(x)

Чтобы решить уравнение с модулем
надо избавиться от модульных
скобок по определению модуля
|a|=
a,
условие1 a ≥ 0
-a,
условие2 a

3. Уравнение вида | f(x)| = g(x)

Условие 1 f(х)≥0 (решаем
полученное неравенство)
2. Раскрываем модульные скобки с
использование условия
f(x)=g(x)
3. Решаем полученное уравнение
4. Проверяем соответствие корней
условию
1.

4. Уравнение вида | f(x)| = g(x)

Условие 2 f(х)
полученное неравенство)
2. Раскрываем модульные скобки с
использование условия
-f(x)=g(x)
3. Решаем полученное уравнение
4. Проверяем соответствие корней
условию
1.

5. Решить уравнение |2x+5|=3x-1

1. Условие: 2x+5≥0
x≥-2,5
Раскрываем модульные скобки: по условию
выражение под модулем положительно,
то модульные скобки просто убираем
2x+5=3x-1
2х-3х=-1-5
-x=-6
X=6 – подходит по условию, следовательно
корень

6. Решить уравнение |2x+5|=3x-1

1. Условие: 2x+5
x
Раскрываем модульные скобки: по условию
выражение под модулем отрицательно, то
модульные скобки раскрываем со знаком
минус
-(2x+5)=3x-1
-2x-5=3x-1
-2х-3х=-1+5
-5x=4
X=-0,8 – не подходит по условию,
следовательно не корень
Ответ: 6

7. Неравенство вида | f(x)| ≥ g(x)

Решаем аналогично уравнению.
1.
Ставим условие 1 и решаем его
2.
Раскрываем модульные скобки в
соответствии с условием
3.
Решаем полученное неравенство
4.
Находим общее решение для условия и
решенного неравенства
5.
Ставим условие 2 и выполняем пункты
со 2 по 4
6.
Объединяем все полученные
промежутки

8. Решить уравнение |2x+5|>3x-1

Решить уравнение |2x+5|>3x-1
1. Условие: 2x+5≥0
x≥-2,5
Раскрываем модульные скобки: по условию
выражение под модулем положительно, то
модульные скобки просто убираем
2x+5>3x-1
2х-3х>-1-5
-x>-6
X
[-2,5;6)
-2,5
6

9. Решить уравнение |2x+5|>3x-1

Решить уравнение |2x+5|>3x-1
2. Условие: 2x+5
x
Раскрываем модульные скобки: по условию
выражение под модулем отрицательно, то
модульные скобки раскрываем с минусом
-(2x+5)>3x-1
-2х-5>3х-1
-2х-3х>-1+5
-5x>4
Х
-2,5
(-∞;-2,5)
-0,8

10. Решить уравнение |2x+5|>3x-1

Решить уравнение |2x+5|>3x-1
Объединим полученные интервалы
(-∞;-2,5)и [-2,5;6)
-2,5
Ответ: (-∞;6)
6

11. Уравнение вида | f(x)| =| g(x) |

| f(x)| =| g(x) |
Заменяем модульные скобки
квадратами
f(x)2 = g(x)2
f(x)2 — g(x)2=0
(f(x) — g(x))(f(x) +g(x)) =0
f(x) — g(x)=0 или f(x) +g(x) =0

12. Решить уравнение |2x+5|=|3x-1|

|2x+5|=|3x-1|
(2x+5)2 =(3x-1)2
(2x+5)2 -(3x-1)2 =0
((2x+5)-(3x-1))((2x+5)+(3x-1))=0
(2x+5)-(3x-1)=0 или (2x+5)+(3x-1)=0
2x+5-3x+1=0
2x+5+3x-1=0
-x=-6
5x=-4
X=6
x=-0,8
Ответ: 6; -0,8

Как решать модули?

Модуль – это абсолютная величина выражения. Чтобы хоть как-то обозначить модуль, принято использовать прямые скобки. То значение, которое заключено в ровных скобках, и является тем значением, которое взято по модулю. Процесс решения любого модуля заключается в раскрытии тех самых прямых скобок, которые математическим языком именуются модульными скобками. Их раскрытие происходит по определенному ряду правил. Также, в порядке решения модулей, находятся и множества значений тех выражений, которые находились в модульных скобках. В большей части всех случаев, модуль раскрывается таким способом, что выражение, которое было подмодульным, получает и положительные, и отрицательные значения, в числе которых также и значение ноль. Если отталкиваться от установленных свойств модуля, то в процессе составляются различные уравнения  или же неравенства от исходного выражения, которые затем необходимо решить. Разберемся же с тем, как решать модули.

Процесс решения

Решение модуля начинается с записи исходного уравнения с модулем. Чтобы ответить на вопрос о том, как решать уравнения с модулем, нужно раскрыть его полностью. Для решения такого уравнения, модуль раскрывается. Все модульные выражения должны быть рассмотрены. Следует определить при каких значениях неизвестных величин, входящих в его состав, модульное выражение в скобках обращается в ноль. Для того чтобы это сделать, достаточно приравнять выражение в модульных скобках к нулю, а затем высчитать решение образовавшегося уравнения. Найденные значения нужно зафиксировать. Таким же способом нужно определить еще и значение всех неизвестных переменных для всех модулей в данном уравнении. Далее необходимо заняться определением и рассмотрением всех случаев существования переменных в выражениях, когда они отличны от значения ноль. Для этого нужно записать некоторую систему из неравенств  соответственно всем модулям в исходном неравенстве. Неравенства должны быть составлены так, чтоб они охватывали все имеющиеся и возможные значения для переменной, которые находят на числовой прямой. Затем нужно начертить для визуализации эту самую числовую прямую, на которой в дальнейшем отложить все полученные значения.

Практически все сейчас можно сделать в интернете. Не является исключением из правил и модуль. Решить онлайн его можно на одном из многочисленных современных ресурсов. Все те значения переменной, которые находятся в нулевом модуле, будут особым ограничением, которое будет использовано в процессе решения модульного уравнения. В исходном уравнении требуется раскрыть все имеющиеся модульные скобки, при этом, изменяя знак выражения, таким образом, чтобы значения искомой переменной совпадали с теми значениями, которые видно на числовой прямой. Полученное уравнение необходимо решить. То значение переменной, которое будет получено в ходе решения уравнения, нужно проверять на ограничение, которое задано самим модулем. Если значение переменной полностью удовлетворяет условие, то оно является правильным. Все корни, которые будут получены в ходе решения уравнения, но не будут подходить по ограничениям, должны быть отброшены.

Пример решения:

Уравнения с модулем и корнем примеры решения. Модуль числа (абсолютная величина числа), определения, примеры, свойства. Защита персональной информации

Модуль числа легко найти, и теория, которая лежит в его основе, важна при решении задач.

Свойства и правила раскрытия, используемые при решении упражнений и на экзаменах, будут полезны школьникам и студентам. Заработай деньги с помощью своих знаний на https://teachs.ru !

Что такое модуль в математике

Модуль числа описывает расстояние на числовой линии от нуля до точки без учета того, в каком направлении от нуля лежит точка. Математическое обозначение: |x|.

Иными словами, это абсолютная величина числа. Определение доказывает, что значение никогда не бывает отрицательным.

Свойства модуля

Важно помнить о следующих свойствах:

Модуль комплексного числа

Абсолютной величиной комплексного числа называют длину направленного отрезка, проведенного от начала комплексной плоскости до точки (a, b).

Этот направленный отрезок также является вектором, представляющим комплексное число a + bi , поэтому абсолютная величина комплексного числа – это то же самое, что и величина (или длина) вектора, представляющего a+ bi .

Как решать уравнения с модулем

Уравнение с модулем – это равенство, которое содержит выражение абсолютного значения. Если для действительного числа оно представляет его расстояние от начала координат на числовой линии, то неравенства с модулем являются типом неравенств, которые состоят из абсолютных значений.

Уравнения типа |x| = a

Уравнение |x| = a имеет два ответа x = a и x = –a , потому что оба варианта находятся на координатной прямой на расстоянии a от 0.

Равенство с абсолютной величиной не имеет решения, если величина отрицательная.

Если |x|

Уравнения типа |x| = |y|

Когда есть абсолютные значения по обе стороны уравнений, нужно рассмотреть обе возможности для приемлемых определений – положительные и отрицательные выражения.

Например, для равенства |x − a| = |x + b| есть два варианта: (x − a) = − (x + b) или (x − a) = (x + b).

Уравнения типа |x| = y

Уравнения такого вида содержат абсолютную величину выражения с переменной слева от нуля, а справа – еще одну неизвестную. Переменная y может быть как больше, так и меньше нуля.

Для получения ответа в таком равенстве нужно решить систему из нескольких уравнений, в которой нужно убедиться, что y – неотрицательная величина:

Решение неравенств с модулем

Чтобы лучше понять, как раскрыть модуль в разных типах равенств и неравенств, нужно проанализировать примеры.

Уравнения вида |x| = a

Пример 1 (алгебра 6 класс). Решить: |x| + 2 = 4.

Решение.

Такие уравнения решаются так же, как и равенства без абсолютных значений. Это означает, что, перемещая неизвестные влево, а константы – вправо, выражение не меняется.

После перемещения константы вправо получено: |x| = 2 .

Поскольку неизвестные связаны с абсолютным значением, это равенство имеет два ответа: 2 и −2 .

Ответ: 2 и −2 .

Пример 2 (алгебра 7 класс). Решить неравенство |x + 2| ≥ 1.

Решение.

Первое, что нужно сделать, это найти точки, где абсолютное значение изменится. Для этого выражение приравнивается к 0 . Получено: x = –2 .

Это означает, что –2 – поворотная точка.

Разделим интервал на 2 части:

  1. для x + 2 ≥ 0

[−1; + ∞).

  1. для х + 2

Общим ответом для этих двух неравенств является интервал (−∞; –3].

Окончательное решение объединение ответов отдельных частей:

x (–∞; –3] [–1; + ∞).

Ответ: x (–∞; –3] [–1; + ∞) .

Уравнения вида |x| = |y|

Пример 1 (алгебра 8 класс). Решить уравнение с двумя модулями: 2 * |x – 1| + 3 = 9 – |x – 1|.

Решение:

Ответ: x 1 = 3; x 2 = 1.

Пример 2 (алгебра 8 класс). Решить неравенство:

Решение:

Уравнения вида |x| = y

Пример 1 (алгебра 10 класс). Найти x:

Решение:

Очень важно провести проверку правой части, иначе можно написать в ответ ошибочные корни. Из системы видно, что не лежит в промежутке.

Ответ: x = 0 .

Модуль суммы

Модуль разности

Абсолютная величина разности двух чисел x и y равна расстоянию между точками с координатами X и Y на координатной прямой.

Пример 1.

Пример 2.

Модуль отрицательного числа

Для нахождения абсолютного значения числа, которое меньше нуля, нужно узнать, как далеко оно расположено от нуля. Поскольку расстояние всегда является положительным (невозможно пройти «отрицательные» шаги, это просто шаги в другом направлении), результат всегда положительный. То есть,

Проще говоря, абсолютная величина отрицательного числа имеет противоположное значение.

Модуль нуля

Известно свойство:

Вот почему нельзя сказать, что абсолютная величина – положительное число: ноль не является ни отрицательным, ни положительным.

Модуль в квадрате

Модуль в квадрате всегда равен выражению в квадрате:

Примеры графиков с модулем

Часто в тестах и на экзаменах встречаются задания, которые возможно решить, лишь проанализировав графики. Рассмотрим такие задания.

Пример 1.

Дана функция f(x) = |x|. Необходимо построить график от – 3 до 3 с шагом 1.

Решение:

Объяснение : из рисунка видно, что график симметричен относительно оси Y.

Пример 2 . Необходимо нарисовать и сравнить графики функций f(x) = |x–2| и g(x) = |x|–2.

Решение:

Объяснение: константа внутри абсолютной величины перемещает весь график вправо, если ее значение отрицательное, и влево, если положительное. Но постоянная снаружи будет передвигать график вверх, если значение положительное, и вниз, если оно отрицательное (как –2 в функции g (x)) .

Координата вершины x (точка, в которой соединяются две линии, вершина графа) – это число, на которое график сдвигается влево или вправо. А координата y – это значение, на которое график сдвигается вверх или вниз.

Строить такие графики можно с помощью онлайн приложений для построения. С их помощью можно наглядно посмотреть, как константы влияют на функции.

Метод интервалов в задачах с модулем

Метод интервалов – один из лучших способов найти ответ в задачах с модулем, особенно если в выражении их несколько.

Для использования метода нужно совершить следующие действия:

  1. Приравнять каждое выражение к нулю.
  2. Найти значения переменных.
  3. Нанести на числовую прямую точки, полученные в пункте 2.
  4. Определить на промежутках знак выражений (отрицательное или положительное значение) и нарисовать символ – или + соответственно. Проще всего определить знак с помощью метода подстановки (подставив любое значение из промежутка).
  5. Решить неравенства с полученными знаками.

Пример 1 . Решить методом интервалов.

Решение:

Инструкция

Если модуль представлен в виде непрерывной функции, то значение ее аргумента может быть как положительным, так и отрицательным: |х| = х, х ≥ 0; |х| = — х, х

Модуль нулю, а модуль любого положительного числа – ему . Если аргумент отрицательный, то после раскрытия скобок его знак меняется с минуса на плюс. На основании этого вытекает вывод, что модули противоположных равны: |-х| = |х| = х.

Модуль комплексного числа находится по формуле: |a| = √b ² + c ², а |a + b| ≤ |a| + |b|. Если в аргументе присутствует в виде множителя положительное число, то его можно вынести за знак скобки, например: |4*b| = 4*|b|.

Если аргумент представлен в виде сложного числа, то для удобства вычислений допускается порядка членов выражения, заключенного в прямоугольные скобки: |2-3| = |3-2| = 3-2 = 1, поскольку (2-3) меньше нуля.

Возведенный в степень аргумент одновременно находится под знаком корня того же порядка – он решается при помощи : √a² = |a| = ±a.

Если перед вами задача, в которой не указано условие раскрытия скобок модуля, то избавляться от них не нужно – это и будет конечный результат. А если требуется их раскрыть, то необходимо указать знак ±. Например, нужно найти значение выражения √(2 * (4-b)) ². Его решение выглядит следующим образом: √(2 * (4-b)) ² = |2 * (4-b)| = 2 * |4-b|. Поскольку знак выражения 4-b неизвестен, то его нужно оставить в скобках. Если добавить дополнительное условие, например, |4-b| >

Модуль нуля равен нулю, а модуль любого положительного числа – ему самому. Если аргумент отрицательный, то после раскрытия скобок его знак меняется с минуса на плюс. На основании этого вытекает вывод, что модули противоположных чисел равны: |-х| = |х| = х.

Модуль комплексного числа находится по формуле: |a| = √b ² + c ², а |a + b| ≤ |a| + |b|. Если в аргументе присутствует в виде множителя целое положительное число, то его можно вынести за знак скобки, например: |4*b| = 4*|b|.

Отрицательным модуль быть не может, поэтому любое отрицательное число преобразуется в положительное: |-x| = x, |-2| = 2, |-1/7| = 1/7, |-2,5| = 2,5.

Если аргумент представлен в виде сложного числа, то для удобства вычислений допускается изменение порядка членов выражения, заключенного в прямоугольные скобки: |2-3| = |3-2| = 3-2 = 1, поскольку (2-3) меньше нуля.

Если перед вами задача, в которой не указано условие раскрытия скобок модуля, то избавляться от них не нужно – это и будет конечный результат. А если требуется их раскрыть, то необходимо указать знак ±. Например, нужно найти значение выражения √(2 * (4-b)) ². Его решение выглядит следующим образом: √(2 * (4-b)) ² = |2 * (4-b)| = 2 * |4-b|. Поскольку знак выражения 4-b неизвестен, то его нужно оставить в скобках. Если добавить дополнительное условие, например, |4-b| > 0, то в итоге получится 2 * |4-b| = 2 *(4 — b). В качестве неизвестного элемента также может быть задано конкретное число, которое следует принимать во внимание, т.к. оно будет влиять на знак выражения.

Модуль – это абсолютная величина выражения. Чтобы хоть как-то обозначить модуль, принято использовать прямые скобки. То значение, которое заключено в ровных скобках, и является тем значением, которое взято по модулю. Процесс решения любого модуля заключается в раскрытии тех самых прямых скобок, которые математическим языком именуются модульными скобками. Их раскрытие происходит по определенному ряду правил. Также, в порядке решения модулей, находятся и множества значений тех выражений, которые находились в модульных скобках. В большей части всех случаев, модуль раскрывается таким способом, что выражение, которое было подмодульным, получает и положительные, и отрицательные значения, в числе которых также и значение ноль. Если отталкиваться от установленных свойств модуля, то в процессе составляются различные уравнения или же неравенства от исходного выражения, которые затем необходимо решить. Разберемся же с тем, как решать модули.

Процесс решения

Решение модуля начинается с записи исходного уравнения с модулем. Чтобы ответить на вопрос о том, как решать уравнения с модулем, нужно раскрыть его полностью. Для решения такого уравнения, модуль раскрывается. Все модульные выражения должны быть рассмотрены. Следует определить при каких значениях неизвестных величин, входящих в его состав, модульное выражение в скобках обращается в ноль. Для того чтобы это сделать, достаточно приравнять выражение в модульных скобках к нулю, а затем высчитать решение образовавшегося уравнения. Найденные значения нужно зафиксировать. Таким же способом нужно определить еще и значение всех неизвестных переменных для всех модулей в данном уравнении. Далее необходимо заняться определением и рассмотрением всех случаев существования переменных в выражениях, когда они отличны от значения ноль. Для этого нужно записать некоторую систему из неравенств соответственно всем модулям в исходном неравенстве. Неравенства должны быть составлены так, чтоб они охватывали все имеющиеся и возможные значения для переменной, которые находят на числовой прямой. Затем нужно начертить для визуализации эту самую числовую прямую, на которой в дальнейшем отложить все полученные значения.

Практически все сейчас можно сделать в интернете. Не является исключением из правил и модуль. Решить онлайн его можно на одном из многочисленных современных ресурсов. Все те значения переменной, которые находятся в нулевом модуле, будут особым ограничением, которое будет использовано в процессе решения модульного уравнения. В исходном уравнении требуется раскрыть все имеющиеся модульные скобки, при этом, изменяя знак выражения, таким образом, чтобы значения искомой переменной совпадали с теми значениями, которые видно на числовой прямой. Полученное уравнение необходимо решить. То значение переменной, которое будет получено в ходе решения уравнения, нужно проверять на ограничение, которое задано самим модулем. Если значение переменной полностью удовлетворяет условие, то оно является правильным. Все корни, которые будут получены в ходе решения уравнения, но не будут подходить по ограничениям, должны быть отброшены.

Термин (module) в буквальном переводе с латинского означает «мера». Это понятие было введено в математику английским учёным Р. Котесом. А немецкий математик К. Вейерштрасс ввёл в обращение знак модуля — символ, которым это понятие обозначается при написании.

Вконтакте

Впервые данное понятие изучается в математике по программе 6 класса средней школы. Согласно одному из определений, модуль — это абсолютное значение действительного числа. Другими словами, чтобы узнать модуль действительного числа, необходимо отбросить его знак.

Графически абсолютное значение а обозначается как |a| .

Основная отличительная черта этого понятия заключается в том, что он всегда является неотрицательной величиной.

Числа, которые отличаются друг от друга только знаком, называются противоположными. Если значение положительное, то противоположное ему будет отрицательным, а ноль является противоположным самому себе.

Геометрическое значение

Если рассматривать понятие модуля с позиций геометрии, то он будет обозначать расстояние, которое измеряется в единичных отрезках от начала координат до заданной точки. Это определение полностью раскрывает геометрический смысл изучаемого термина.

Графически это можно выразить следующим образом: |a| = OA.

Свойства абсолютной величины

Ниже будут рассмотрены все математические свойства этого понятия и способы записи в виде буквенных выражений:

Особенности решения уравнений с модулем

Если говорить о решении математических уравнений и неравенств, в которых содержится module, то необходимо помнить, что для их решения потребуется открыть этот знак.

К примеру, если знак абсолютной величины содержит в себе некоторое математическое выражение, то перед тем как раскрыть модуль, необходимо учитывать действующие математические определения.

|А + 5| = А + 5 , если, А больше или равняется нулю.

5-А , если, А значение меньше нуля.

В некоторых случаях знак может раскрываться однозначно при любых значениях переменной.

Рассмотрим ещё одни пример. Построим координатную прямую, на которой отметим все числовые значения абсолютной величиной которых будет 5.

Для начала необходимо начертить координатную прямую, обозначить на ней начало координат и задать размер единичного отрезка. Кроме того, прямая должна иметь направление. Теперь на этой прямой необходимо нанести разметки, которые будут равны величине единичного отрезка.

Таким образом, мы можем увидеть, что на этой координатной прямой будут две интересующие нас точки со значениями 5 и -5.

Одна из самых сложных тем для учащихся – это решение уравнений, содержащих переменную под знаком модуля. Давайте разберемся для начала с чем же это связано? Почему, например, квадратные уравнения большинство детей щелкает как орешки, а с таким далеко не самым сложным понятием как модуль имеет столько проблем?

На мой взгляд, все эти сложности связаны с отсутствием четко сформулированных правил для решения уравнений с модулем. Так, решая квадратное уравнение, ученик точно знает, что ему нужно сначала применять формулу дискриминанта, а затем формулы корней квадратного уравнения. А что делать, если в уравнении встретился модуль? Постараемся четко описать необходимый план действий на случай, когда уравнение содержит неизвестную под знаком модуля. К каждому случаю приведем несколько примеров.

Но для начала вспомним определение модуля . Итак, модулем числа a называется само это число, если a неотрицательно и -a , если число a меньше нуля. Записать это можно так:

|a| = a, если a ≥ 0 и |a| = -a, если a

Говоря о геометрическом смысле модуля, следует помнить, что каждому действительному числу соответствует определенная точка на числовой оси – ее координата. Так вот, модулем или абсолютной величиной числа называется расстояние от этой точки до начала отсчета числовой оси. Расстояние всегда задается положительным числом. Таким образом, модуль любого отрицательного числа есть число положительное. Кстати, даже на этом этапе многие ученики начинают путаться. В модуле может стоять какое угодно число, а вот результат применения модуля всегда число положительное.

Теперь перейдем непосредственно к решению уравнений.

1. Рассмотрим уравнение вида |x| = с, где с – действительное число. Это уравнение можно решить с помощью определения модуля.

Все действительные числа разобьем на три группы: те, что больше нуля, те, что меньше нуля, и третья группа – это число 0. Запишем решение в виде схемы:

{±c, если с > 0

Если |x| = c, то x = {0, если с = 0

{нет корней, если с

1) |x| = 5, т.к. 5 > 0, то x = ±5;

2) |x| = -5, т.к. -5

3) |x| = 0, то x = 0.

2. Уравнение вида |f(x)| = b, где b > 0. Для решения данного уравнения необходимо избавиться от модуля. Делаем это так: f(x) = b или f(x) = -b. Теперь необходимо решить отдельно каждое из полученных уравнений. Если в исходном уравнении b

1) |x + 2| = 4, т.к. 4 > 0, то

x + 2 = 4 или x + 2 = -4

2) |x 2 – 5| = 11, т.к. 11 > 0, то

x 2 – 5 = 11 или x 2 – 5 = -11

x 2 = 16 x 2 = -6

x = ± 4 нет корней

3) |x 2 – 5x| = -8 , т.к. -8

3. Уравнение вида |f(x)| = g(x). По смыслу модуля такое уравнение будет иметь решения, если его правая часть больше или равна нулю, т.е. g(x) ≥ 0. Тогда будем иметь:

f(x) = g(x) или f(x) = -g(x) .

1) |2x – 1| = 5x – 10. Данное уравнение будет иметь корни, если 5x – 10 ≥ 0. Именно с этого и начинают решение таких уравнений.

1. О.Д.З. 5x – 10 ≥ 0

2. Решение:

2x – 1 = 5x – 10 или 2x – 1 = -(5x – 10)

3. Объединяем О.Д.З. и решение, получаем:

Корень x = 11/7 не подходит по О.Д.З., он меньше 2, а x = 3 этому условию удовлетворяет.

Ответ: x = 3

2) |x – 1| = 1 – x 2 .

1. О.Д.З. 1 – x 2 ≥ 0. Решим методом интервалов данное неравенство:

(1 – x)(1 + x) ≥ 0

2. Решение:

x – 1 = 1 – x 2 или x – 1 = -(1 – x 2)

x 2 + x – 2 = 0 x 2 – x = 0

x = -2 или x = 1 x = 0 или x = 1

3. Объединяем решение и О.Д.З.:

Подходят только корни x = 1 и x = 0.

Ответ: x = 0, x = 1.

4. Уравнение вида |f(x)| = |g(x)|. Такое уравнение равносильно двум следующим уравнениям f(x) = g(x) или f(x) = -g(x).

1) |x 2 – 5x + 7| = |2x – 5|. Данное уравнение равносильно двум следующим:

x 2 – 5x + 7 = 2x – 5 или x 2 – 5x +7 = -2x + 5

x 2 – 7x + 12 = 0 x 2 – 3x + 2 = 0

x = 3 или x = 4 x = 2 или x = 1

Ответ: x = 1, x = 2, x = 3, x = 4.

5. Уравнения, решаемые методом подстановки (замены переменной). Данный метод решения проще всего объяснить на конкретном примере. Так, пусть дано квадратное уравнение с модулем:

x 2 – 6|x| + 5 = 0. По свойству модуля x 2 = |x| 2 , поэтому уравнение можно переписать так:

|x| 2 – 6|x| + 5 = 0. Сделаем замену |x| = t ≥ 0, тогда будем иметь:

t 2 – 6t + 5 = 0. Решая данное уравнение, получаем, что t = 1 или t = 5. Вернемся к замене:

|x| = 1 или |x| = 5

x = ±1 x = ± 5

Ответ: x = -5, x = -1, x = 1, x = 5.

Рассмотрим еще один пример:

x 2 + |x| – 2 = 0. По свойству модуля x 2 = |x| 2 , поэтому

|x| 2 + |x| – 2 = 0. Сделаем замену |x| = t ≥ 0, тогда:

t 2 + t – 2 = 0. Решая данное уравнение, получаем, t = -2 или t = 1. Вернемся к замене:

|x| = -2 или |x| = 1

Нет корней x = ± 1

Ответ: x = -1, x = 1.

6. Еще один вид уравнений – уравнения со «сложным» модулем. К таким уравнениям относятся уравнения, в которых есть «модули в модуле». Уравнения данного вида можно решать, применяя свойства модуля.

1) |3 – |x|| = 4. Будем действовать так же, как и в уравнениях второго типа. Т.к. 4 > 0, то получим два уравнения:

3 – |x| = 4 или 3 – |x| = -4.

Теперь выразим в каждом уравнении модуль х, тогда |x| = -1 или |x| = 7.

Решаем каждое из полученных уравнений. В первом уравнении нет корней, т.к. -1

Ответ x = -7, x = 7.

2) |3 + |x + 1|| = 5. Решаем это уравнение аналогичным образом:

3 + |x + 1| = 5 или 3 + |x + 1| = -5

|x + 1| = 2 |x + 1| = -8

x + 1 = 2 или x + 1 = -2. Нет корней.

Ответ: x = -3, x = 1.

Существует еще и универсальный метод решения уравнений с модулем. Это метод интервалов. Но мы его рассмотрим в дальнейшем.

blog.сайт, при полном или частичном копировании материала ссылка на первоисточник обязательна.

Читайте также…
Модульный арифметический решатель

— Калькулятор сравнения

Поиск инструмента

Решатель модульных уравнений

Инструмент / решатель для решения модульного уравнения. Модульное уравнение — это математическое выражение, представленное в форме сравнения по крайней мере с одной неизвестной переменной.

Результаты

Модуль решения модульных уравнений

— dCode

Тэги: Арифметика

Поделиться

dCode и другие

dCode является бесплатным, а его инструменты являются ценным подспорьем в играх, математике, геокешинге, головоломках и задачах, которые нужно решать каждый день!
Предложение? обратная связь? Жук ? идея ? Запись в dCode !

Калькулятор модульных уравнений

Решите уравнения с несколькими модулями

В частном случае одно неизвестное с несколькими уравнениями с несколькими модулями , есть китайская теорема об остатках:

Ответы на вопросы (FAQ)

Что такое модульное уравнение? (Определение)

Модульное уравнение — это уравнение (или система уравнений, по крайней мере, с одной неизвестной переменной), действительное в соответствии с линейным сравнением (по модулю / модулю).С модулем вместо того, чтобы говорить о равенстве, принято говорить о конгруэнтности.

Для системы уравнений с несколькими модулями (нелинейной) это другой расчет, который может быть решен с помощью калькулятора, решающего китайскую проблему остатков, доступную в dCode.

Как решить модульное уравнение?

Как решить несколько уравнений?

Введите по одному уравнению в каждой строке или разделите их оператором &&.

Как написать символ сравнения ≡?

Нет необходимости писать ≡ (конгруэнтно), чтобы dCode мог решать уравнения, достаточно знака равенства =.

Задайте новый вопрос

Исходный код

dCode сохраняет за собой право собственности на исходный код онлайн-инструмента Modular Equation Solver. За исключением явной лицензии с открытым исходным кодом (обозначенной CC / Creative Commons / free), любой алгоритм, апплет или фрагмент «Modular Equation Solver» (конвертер, решатель, шифрование / дешифрование, кодирование / декодирование, шифрование / дешифрование, переводчик) или любой «Modular Equation Solver» Функция Equation Solver (вычисление, преобразование, решение, дешифрование / шифрование, дешифрование / шифрование, декодирование / кодирование, перевод), написанная на любом информатическом языке (Python, Java, PHP, C #, Javascript, Matlab и т. Д.)) и никакая загрузка данных, скрипт, копипаст или доступ к API для «Modular Equation Solver» не будут бесплатными, то же самое для автономного использования на ПК, планшете, iPhone или Android! dCode распространяется бесплатно и онлайн.

Нужна помощь?

Пожалуйста, посетите наше сообщество dCode Discord для запросов о помощи!
NB: для зашифрованных сообщений проверьте наш автоматический идентификатор шифра!

Вопросы / комментарии

Сводка

Похожие страницы

Поддержка

Форум / Справка

Ключевые слова

модульный, модуль, модуль, уравнение, сравнение, конгруэнтность, модуль, равенство, калькулятор

Ссылки


Источник: https: // www.dcode.fr/modular-equation-solver

© 2021 dCode — Идеальный «инструментарий» для решения любых игр / загадок / геокэшинга / CTF. Модульный арифметический решатель

— Калькулятор сравнения

Поиск инструмента

Решатель модульных уравнений

Инструмент / решатель для решения модульного уравнения. Модульное уравнение — это математическое выражение, представленное в форме сравнения по крайней мере с одной неизвестной переменной.

Результаты

Модуль решения модульных уравнений

— dCode

Тэги: Арифметика

Поделиться

dCode и другие

dCode является бесплатным, а его инструменты являются ценным подспорьем в играх, математике, геокешинге, головоломках и задачах, которые нужно решать каждый день!
Предложение? обратная связь? Жук ? идея ? Запись в dCode !

Калькулятор модульных уравнений

Решите уравнения с несколькими модулями

В частном случае одно неизвестное с несколькими уравнениями с несколькими модулями , есть китайская теорема об остатках:

Ответы на вопросы (FAQ)

Что такое модульное уравнение? (Определение)

Модульное уравнение — это уравнение (или система уравнений, по крайней мере, с одной неизвестной переменной), действительное в соответствии с линейным сравнением (по модулю / модулю).С модулем вместо того, чтобы говорить о равенстве, принято говорить о конгруэнтности.

Для системы уравнений с несколькими модулями (нелинейной) это другой расчет, который может быть решен с помощью калькулятора, решающего китайскую проблему остатков, доступную в dCode.

Как решить модульное уравнение?

Как решить несколько уравнений?

Введите по одному уравнению в каждой строке или разделите их оператором &&.

Как написать символ сравнения ≡?

Нет необходимости писать ≡ (конгруэнтно), чтобы dCode мог решать уравнения, достаточно знака равенства =.

Задайте новый вопрос

Исходный код

dCode сохраняет за собой право собственности на исходный код онлайн-инструмента Modular Equation Solver. За исключением явной лицензии с открытым исходным кодом (обозначенной CC / Creative Commons / free), любой алгоритм, апплет или фрагмент «Modular Equation Solver» (конвертер, решатель, шифрование / дешифрование, кодирование / декодирование, шифрование / дешифрование, переводчик) или любой «Modular Equation Solver» Функция Equation Solver (вычисление, преобразование, решение, дешифрование / шифрование, дешифрование / шифрование, декодирование / кодирование, перевод), написанная на любом информатическом языке (Python, Java, PHP, C #, Javascript, Matlab и т. Д.)) и никакая загрузка данных, скрипт, копипаст или доступ к API для «Modular Equation Solver» не будут бесплатными, то же самое для автономного использования на ПК, планшете, iPhone или Android! dCode распространяется бесплатно и онлайн.

Нужна помощь?

Пожалуйста, посетите наше сообщество dCode Discord для запросов о помощи!
NB: для зашифрованных сообщений проверьте наш автоматический идентификатор шифра!

Вопросы / комментарии

Сводка

Похожие страницы

Поддержка

Форум / Справка

Ключевые слова

модульный, модуль, модуль, уравнение, сравнение, конгруэнтность, модуль, равенство, калькулятор

Ссылки


Источник: https: // www.dcode.fr/modular-equation-solver

© 2021 dCode — Идеальный «инструментарий» для решения любых игр / загадок / геокэшинга / CTF. Модульный арифметический решатель

— Калькулятор сравнения

Поиск инструмента

Решатель модульных уравнений

Инструмент / решатель для решения модульного уравнения. Модульное уравнение — это математическое выражение, представленное в форме сравнения по крайней мере с одной неизвестной переменной.

Результаты

Модуль решения модульных уравнений

— dCode

Тэги: Арифметика

Поделиться

dCode и другие

dCode является бесплатным, а его инструменты являются ценным подспорьем в играх, математике, геокешинге, головоломках и задачах, которые нужно решать каждый день!
Предложение? обратная связь? Жук ? идея ? Запись в dCode !

Калькулятор модульных уравнений

Решите уравнения с несколькими модулями

В частном случае одно неизвестное с несколькими уравнениями с несколькими модулями , есть китайская теорема об остатках:

Ответы на вопросы (FAQ)

Что такое модульное уравнение? (Определение)

Модульное уравнение — это уравнение (или система уравнений, по крайней мере, с одной неизвестной переменной), действительное в соответствии с линейным сравнением (по модулю / модулю).С модулем вместо того, чтобы говорить о равенстве, принято говорить о конгруэнтности.

Для системы уравнений с несколькими модулями (нелинейной) это другой расчет, который может быть решен с помощью калькулятора, решающего китайскую проблему остатков, доступную в dCode.

Как решить модульное уравнение?

Как решить несколько уравнений?

Введите по одному уравнению в каждой строке или разделите их оператором &&.

Как написать символ сравнения ≡?

Нет необходимости писать ≡ (конгруэнтно), чтобы dCode мог решать уравнения, достаточно знака равенства =.

Задайте новый вопрос

Исходный код

dCode сохраняет за собой право собственности на исходный код онлайн-инструмента Modular Equation Solver. За исключением явной лицензии с открытым исходным кодом (обозначенной CC / Creative Commons / free), любой алгоритм, апплет или фрагмент «Modular Equation Solver» (конвертер, решатель, шифрование / дешифрование, кодирование / декодирование, шифрование / дешифрование, переводчик) или любой «Modular Equation Solver» Функция Equation Solver (вычисление, преобразование, решение, дешифрование / шифрование, дешифрование / шифрование, декодирование / кодирование, перевод), написанная на любом информатическом языке (Python, Java, PHP, C #, Javascript, Matlab и т. Д.)) и никакая загрузка данных, скрипт, копипаст или доступ к API для «Modular Equation Solver» не будут бесплатными, то же самое для автономного использования на ПК, планшете, iPhone или Android! dCode распространяется бесплатно и онлайн.

Нужна помощь?

Пожалуйста, посетите наше сообщество dCode Discord для запросов о помощи!
NB: для зашифрованных сообщений проверьте наш автоматический идентификатор шифра!

Вопросы / комментарии

Сводка

Похожие страницы

Поддержка

Форум / Справка

Ключевые слова

модульный, модуль, модуль, уравнение, сравнение, конгруэнтность, модуль, равенство, калькулятор

Ссылки


Источник: https: // www.dcode.fr/modular-equation-solver

© 2021 dCode — Идеальный «инструментарий» для решения любых игр / загадок / геокэшинга / CTF.

Программа для решения квадратных модульных уравнений

Это веб-приложение может решать уравнения вида a⁢x² + bx + c ≡ 0 (mod n) , где неизвестное целое число x находится в диапазоне 0 ≤ x . В частности, он может найти модульные квадратные корни, задав a = -1 , b = 0 , c = число, корень которого мы хотим найти и n = модуль .

Вы можете вводить числа или числовые выражения в поля ввода слева.

Калькулятор принимает числа до 1000 цифр, но обратите внимание, что модуль n должен быть разложен на множители (некоторые большие числа не могут быть разложены на множители за разумный промежуток времени). Механизм факторизации — это тот, который используется в апплете факторизации метода эллиптической кривой, который использует методы ECM и SIQS.

Когда a не равно нулю, количество решений зависит от количества различных простых множителей модуля, поэтому, если модуль имеет много маленьких простых множителей (скажем, более 14), программе может не хватить памяти, и она не покажет никакого решения.или ** для возведения в степень (показатель степени должен быть больше или равен нулю).

  • <, == , > ; <= , > = ,! = Для сравнения. Операторы возвращают ноль для false и -1 для true.
  • И , ИЛИ , XOR , НЕ для двоичной логики. Операции выполняются в двоичном формате (основание 2). К положительным (отрицательным) числам добавляется бесконечное количество битов, равных нулю (единице).
  • SHL : Когда b ≥ 0, a SHL b сдвиги a оставили количество битов, указанное в b . Это эквивалентно a × 2 b . В противном случае, a SHL b сдвигает a вправо на количество битов, указанное в — b . Это эквивалентно полу ( a /2 b ). Пример: 5 ШЛ 3 = 40.
  • SHR : Когда b ≥ 0, a SHR b сдвигает a вправо на количество битов, указанное в b .Это эквивалентно полу ( a /2 b ). В противном случае, a SHR b сдвигает a влево на количество битов, указанное в — b . Это эквивалентно a × 2 b . Пример: -19 SHR 2 = -5.
  • н! : факториал ( n должно быть больше или равно нулю). Пример: 6! = 6 × 5 × 4 × 3 × 2 = 720.
  • p # : примитив (произведение всех простых чисел, меньших или равных p ).Пример: 12 # = 11 × 7 × 5 × 3 × 2 = 2310.
  • B (n) : Предыдущее вероятное простое число до n . Пример: B (24) = 23.
  • F (n) : число Фибоначчи F n из последовательности 0, 1, 1, 2, 3, 5, 8, 13, 21 и т. Д., Где каждый элемент равен сумме двух предыдущих членов последовательность. Пример: F (7) = 13.
  • L (n) : число Люка L n = F n -1 + F n +1
  • N (n) : Следующее возможное простое число после n .Пример: N (24) = 29.
  • P (n) : Неограниченный номер раздела (количество разложений n на суммы целых чисел без учета порядка). Пример: P (4) = 5, потому что число 4 можно разделить 5 различными способами: 4 = 3 + 1 = 2 + 2 = 2 + 1 + 1 = 1 + 1 + 1 + 1.
  • Gcd (m, n) : наибольший общий делитель этих двух целых чисел. Пример: GCD (12, 16) = 4.
  • Modinv (m, n) : инверсия m по модулю n , действительно только тогда, когда m и n взаимно просты, что означает, что у них нет общих множителей.Пример: Modinv (3,7) = 5, потому что 3 × 5 ≡ 1 (mod 7)
  • Modpow (m, n, r) : находит m n по модулю r . Пример: Modpow (3, 4, 7) = 4, потому что 3 4 ≡ 4 (mod 7).
  • IsPrime (n) : возвращает ноль, если n не является вероятным простым числом, -1, если это так. Пример: IsPrime (5) = -1.
  • NumDigits (n, r) : количество цифр n в базе r . Пример: NumDigits (13, 2) = 4, потому что 13 в двоичном формате (основание 2) выражается как 1101.
  • SumDigits (n, r) : сумма цифр n по основанию r . Пример: SumDigits (213, 10) = 6, потому что сумма цифр, выраженная в десятичном формате, равна 2 + 1 + 3 = 6.
  • RevDigits (n, r) : находит значение, полученное путем обратной записи цифр n по основанию r . Пример: RevDigits (213, 10) = 312.
  • Вы можете использовать префикс 0x для шестнадцатеричных чисел, например, 0x38 равно 56.

    Символ возведения в степень отсутствует на некоторых мобильных устройствах, поэтому две звездочки ** можно ввести в качестве оператора возведения в степень.

    Исходный код

    Вы можете загрузить исходный код текущей программы и старый апплет квадратного модульного уравнения с GitHub. Обратите внимание, что исходный код написан на языке C, и вам нужна среда Emscripten для создания Javascript.

    Автор Дарио Альперн. Последнее обновление 31 мая 2021 г.

    python — Как изменить мой код для решения модульных уравнений с делением?

    Я написал расширенный алгоритм Евклида, но у меня возникли проблемы с его использованием для решения следующего уравнения:

    , который решатель модульных уравнений сокращает до

    и

    Вот мой код:

      def fastlinearcongruenceSO (powx, divmodx, N, withstats = False):
     x, y, z = egcditerx2SO (powx, N, со статистикой)
     ответ = (y * divmodx)% N
     если withstats == True:
        печать (f "answer = {answer}")
     если x> 1:
        powx // = x
        divmodx // = х
        N // = х
        если withstats == True:
          print (f "powx = {powx}, divmodx = {divmodx}, N = {N}")
        x, y, z = egcditerx2SO (powx, N, со статистикой)
        если withstats == True:
          print (f "x = {x}, y = {y}, z = {z}")
        ответ = (y * divmodx)% N
        если withstats == True:
           печать (f "answer = {answer}")
     ответный ответ
    
    def egcditerx2SO (a, b, withstats = False):
      s = 0
      г = б
      old_s = 1
      old_r = а
      частное = 0
      если withstats == True:
          print (f "частное = {частное}, old_r = {old_r}, r = {r}, old_s = {old_s}, s = {s}")
      а r! = 0:
        quotient = old_r // г
        old_r, r = r, old_r - частное * r
        old_s, s = s, old_s - частное * s
        если withstats == True:
          print (f "частное = {частное}, old_r = {old_r}, r = {r}, old_s = {old_s}, s = {s}")
      если b! = 0:
        bezout_t = quotient = (old_r - old_s * a) // б
        если withstats == True:
          print (f "bezout_t = {bezout_t}")
      еще:
        bezout_t = 0
      если withstats == True:
        print ("Коэффициенты Безу:", (old_s, bezout_t))
        print ("наибольший общий делитель:", old_r)
      вернуть old_r, old_s, bezout_t
      

    Чтобы найти обратные, я использовал эту форму:

      IN: быстрое линейное совпадение SO (327, 1, 1009)
    ВЫХОД: 108
      

    Я не уверен, как или какие изменения мне нужно внести, чтобы изменить решение с помощью формы разделения, есть ли у кого-нибудь идеи о том, какие изменения мне нужно внести, или могу ли я решить эту проблему, используя свой существующий код? Я действительно хочу изменить свой код, чтобы решить это уравнение: 743360 / 1008x = 272 (mod 1009) и x == 116 или чтобы кто-то сообщил мне, как было решено это уравнение, и я могу затем с эти шаги изменяют мой код для решения уравнений модуля на основе деления.Спасибо всем, кто знает, как решать эти уравнения, основанные на делении.

    % PDF-1.6 % 186 0 obj> эндобдж xref 186 95 0000000016 00000 н. 0000003339 00000 н. 0000003516 00000 н. 0000003644 00000 п. 0000003701 00000 п. 0000004304 00000 н. 0000004429 00000 н. 0000004552 00000 н. 0000004676 00000 н. 0000004803 00000 н. 0000004927 00000 н. 0000005054 00000 н. 0000006233 00000 н. 0000006611 00000 н. 0000006984 00000 н. 0000007181 00000 п. 0000008220 00000 н. 0000009397 00000 н. 0000017753 00000 п. 0000018932 00000 п. 0000019956 00000 п. 0000020954 00000 п. 0000021151 00000 п. 0000021548 00000 п. 0000021906 00000 п. 0000031430 00000 п. 0000031485 00000 п. 0000031552 00000 п. 0000031745 00000 п. 0000032765 00000 п. 0000033633 00000 п. 0000034492 00000 п. 0000035535 00000 п. 0000035719 00000 п. 0000036568 00000 п. 0000052190 00000 п. 0000052542 00000 п. 0000052672 00000 п. 0000053847 00000 п. 0000055023 00000 п. 0000055219 00000 п. 0000056191 00000 п. 0000057368 00000 п. 0000057557 00000 п. 0000073173 00000 п. 0000073372 00000 п. 0000073767 00000 п. 0000086689 00000 п. 0000086881 00000 п. 0000087262 00000 п. 0000087470 00000 п. 0000087640 00000 п. 0000088433 00000 п. 0000089607 00000 п. 00000

    00000 п. 00000

    00000 п. 0000102639 00000 н. 0000102831 00000 н. 0000103306 00000 н. 0000103642 00000 п. 0000103927 00000 н. 0000104006 00000 п. 0000104186 00000 п. 0000104824 00000 н. 0000105999 00000 н. 0000106591 00000 н. 0000106636 00000 н. 0000107482 00000 н. 0000107676 00000 н. 0000108406 00000 п. 0000109272 00000 н. 0000109461 00000 п. 0000114031 00000 н. 0000117164 00000 н. 0000131074 00000 н. 0000131255 00000 н. 0000131541 00000 н. 0000131604 00000 н. 0000136686 00000 н. 0000136870 00000 н. 0000140945 00000 н. 0000141129 00000 н. 0000155245 00000 н. 0000155433 00000 н. 0000155718 00000 н. 0000157698 00000 н. 0000157879 00000 н. 0000157935 00000 н. 0000158133 00000 н. 0000158234 00000 н. 0000158282 00000 н. 0000158441 00000 н. 0000158633 00000 н. 0000158801 00000 н. 0000002196 00000 н. трейлер ] >> startxref 0 %% EOF 280 0 obj> поток xb«f`te`g`fb @

    % PDF-1.2 % 89 0 объект > эндобдж xref 89 333 0000000016 00000 н. 0000007009 00000 н. 0000009935 00000 н. 0000010150 00000 п. 0000010393 00000 п. 0000010779 00000 п. 0000010957 00000 п. 0000011305 00000 п. 0000011639 00000 п. 0000011931 00000 п. 0000012245 00000 п. 0000012595 00000 п. 0000012922 00000 п. 0000013251 00000 п. 0000013513 00000 п. 0000013834 00000 п. 0000014105 00000 п. 0000014402 00000 п. 0000014716 00000 п. 0000015123 00000 п. 0000015520 00000 н. 0000015817 00000 п. 0000016158 00000 п. 0000016528 00000 п. 0000016890 00000 н. 0000017218 00000 п. 0000017501 00000 п. 0000017841 00000 п. 0000018133 00000 п. 0000018453 00000 п. 0000018834 00000 п. 0000019196 00000 п. 0000019500 00000 н. 0000019846 00000 п. 0000020123 00000 п. 0000020330 00000 п. 0000020565 00000 п. 0000020780 00000 п. 0000020948 00000 п. 0000021118 00000 п. 0000021453 00000 п. 0000021736 00000 п. 0000022101 00000 п. 0000022447 00000 п. 0000022791 00000 п. 0000023124 00000 п. 0000023400 00000 п. 0000023719 00000 п. 0000023910 00000 п. 0000024259 00000 п. 0000024526 00000 п. 0000024578 00000 п. 0000024799 00000 п. 0000025285 00000 п. 0000025782 00000 п. 0000026041 00000 п. 0000026331 00000 п. 0000026619 00000 п. 0000026872 00000 н. 0000027088 00000 п. 0000027328 00000 п. 0000027626 00000 н. 0000027883 00000 п. 0000028136 00000 п. 0000028402 00000 п. 0000028700 00000 п. 0000028969 00000 п. 0000029089 00000 н. 0000029263 00000 п. 0000029446 00000 н. 0000029658 00000 п. 0000029825 00000 п. 0000030003 00000 п. 0000030276 00000 п. 0000030528 00000 п. 0000030777 00000 п. 0000031046 00000 п. 0000031098 00000 п. 0000031289 00000 п. 0000031522 00000 п. 0000031775 00000 п. 0000031827 00000 н. 0000031936 00000 п. 0000032189 00000 п. 0000032379 00000 п. 0000032558 00000 п. 0000032707 00000 п. 0000032985 00000 п. 0000033263 00000 п. 0000033542 00000 п. 0000033822 00000 п. 0000034071 00000 п. 0000034246 00000 п. 0000034612 00000 п. 0000034664 00000 п. 0000034869 00000 п. 0000035164 00000 п. 0000035409 00000 п. 0000035715 00000 п. 0000035952 00000 п. 0000036167 00000 п. 0000036421 00000 п. 0000036656 00000 п. 0000036826 00000 п. 0000037156 00000 п. 0000037491 00000 п. 0000037818 00000 п. 0000038079 00000 п. 0000038384 00000 п. 0000038718 00000 п. 0000038936 00000 п. 0000039390 00000 н. 0000039442 00000 п. 0000039581 00000 п. 0000039877 00000 п. 0000040179 00000 п. 0000040417 00000 п. 0000040705 00000 п. 0000040979 00000 п. 0000041229 00000 п. 0000041531 00000 п. 0000041790 00000 п. 0000042119 00000 п. 0000042366 00000 п. 0000042589 00000 п. 0000042765 00000 н. 0000043045 00000 п. 0000043291 00000 п. 0000043658 00000 п. 0000044013 00000 п. 0000044347 00000 п. 0000044715 00000 п. 0000045148 00000 п. 0000045513 00000 п. 0000045877 00000 п. 0000046141 00000 п. 0000046448 00000 н. 0000046626 00000 п. 0000046826 00000 п. 0000046994 00000 п. 0000047268 00000 н. 0000047568 00000 п. 0000047870 00000 п. 0000048198 00000 п. 0000048562 00000 н. 0000048881 00000 п. 0000049262 00000 п. 0000049448 00000 п. 0000049786 00000 п. 0000050119 00000 п. 0000050481 00000 п. 0000050717 00000 п. 0000051014 00000 п. 0000051244 00000 п. 0000051531 00000 п. 0000051711 00000 п. 0000051988 00000 п. 0000052225 00000 п. 0000052521 00000 п. 0000053198 00000 п. 0000053649 00000 п. 0000053987 00000 п. 0000054232 00000 п. 0000054474 00000 п. 0000054827 00000 н. 0000055118 00000 п. 0000055430 00000 п. 0000055728 00000 п. 0000056074 00000 п. 0000056353 00000 п. 0000056727 00000 н. 0000057207 00000 п. 0000057489 00000 п. 0000057733 00000 п. 0000057936 00000 п. 0000058201 00000 п. 0000058561 00000 п. 0000058859 00000 п. 0000059157 00000 п. 0000059209 00000 п. 0000059456 00000 п. 0000059813 00000 п. 0000060043 00000 п. 0000060386 00000 п. 0000060731 00000 п. 0000061079 00000 п. 0000061347 00000 п. 0000061634 00000 п. 0000061805 00000 п. 0000062062 00000 н. 0000062238 00000 п. 0000062588 00000 п. 0000063001 00000 п. 0000063344 00000 п. 0000063682 00000 п. 0000064058 00000 п. 0000064356 00000 п. 0000064666 00000 п. 0000064851 00000 п. 0000065134 00000 п. 0000065479 00000 п. 0000065755 00000 п. 0000066011 00000 п. 0000066376 00000 п. 0000066602 00000 п. 0000066788 00000 п. 0000067043 00000 п. 0000067422 00000 п. 0000067634 00000 п. 0000067960 00000 п. 0000068193 00000 п. 0000068434 00000 п. 0000068716 00000 п. 0000069004 00000 п. 0000069276 00000 п. 0000069503 00000 п. 0000069777 00000 п. 0000070779 00000 п. 0000071315 00000 п. 0000071601 00000 п. 0000071940 00000 п. 0000072259 00000 п. 0000072578 00000 п. 0000072880 00000 п. 0000073225 00000 п. 0000073453 00000 п. 0000073746 00000 п. 0000074050 00000 п. 0000074344 00000 п. 0000074617 00000 п. 0000074899 00000 н. 0000075082 00000 п. 0000075361 00000 п. 0000075659 00000 п. 0000075902 00000 п. 0000076105 00000 п. 0000076349 00000 п. 0000076590 00000 п. 0000076612 00000 п. 0000077477 00000 п. 0000077704 00000 п. 0000077934 00000 п. 0000078107 00000 п. 0000078293 00000 п. 0000078345 00000 п. 0000078439 00000 п. 0000078540 00000 п. 0000078796 00000 п. 0000079042 00000 п. 0000079275 00000 п. 0000079297 00000 п. 0000080195 00000 п. 0000080555 00000 п. 0000080906 00000 п. 0000081293 00000 п. 0000081616 00000 п. 0000081985 00000 п. 0000082306 00000 п. 0000082699 00000 н. 0000082985 00000 п. 0000083350 00000 п. 0000083673 00000 п. 0000084044 00000 п. 0000084276 00000 п. 0000084573 00000 п. 0000084995 00000 п. 0000085346 00000 п. 0000085563 00000 п. 0000085740 00000 п. 0000086041 00000 п. 0000086093 00000 п. 0000086367 00000 п. 0000086672 00000 н. 0000086931 00000 п. 0000087304 00000 п. 0000087497 00000 п. 0000087845 00000 п. 0000088231 00000 п. 0000088568 00000 п. 0000088916 00000 п. 0000089323 00000 п. 0000089666 00000 п. 00000

    00000 п. 00000
  • 00000 н. 00000 00000 п. 0000090922 00000 н. 0000091244 00000 п. 0000091532 00000 п. 0000092185 00000 п. 0000092630 00000 п. 0000092981 00000 п. 0000093244 00000 п. 0000093564 00000 п. 0000093862 00000 п. 0000094127 00000 п. 0000094433 00000 п. 0000094745 00000 п. 0000095098 00000 п. 0000095375 00000 п. 0000095690 00000 н. 0000096030 00000 н. 0000096395 00000 п. 0000096720 00000 н. 0000096996 00000 н. 0000097332 00000 п. 0000097623 00000 п. 0000097958 00000 п. 0000098276 00000 п. 0000098298 00000 п. 0000099112 00000 н. 0000099134 00000 п. 0000099902 00000 н. 0000100125 00000 н. 0000100393 00000 н. 0000100611 00000 н. 0000100663 00000 н. 0000100925 00000 н. 0000101018 00000 н. 0000101115 00000 н. 0000101357 00000 н. 0000101679 00000 п. 0000101922 00000 н. 0000101944 00000 н. 0000102773 00000 н. 0000102795 00000 н. 0000103657 00000 п. 0000103679 00000 п. 0000104515 00000 н. 0000104719 00000 н. 0000104741 00000 н. 0000105531 00000 н. 0000007064 00000 н. 0000009912 00000 н. трейлер ] >> startxref 0 %% EOF 90 0 объект > эндобдж 420 0 объект > поток HWhCB @ H ҫ0 0I! «C @ j! @ (Gp (tT @ Efgf9 {v {ソ}

    .
  • Добавить комментарий

    Ваш адрес email не будет опубликован. Обязательные поля помечены *